Understanding business 8th edition, Business Management Mid-term (Study Guide Q&A), Business 101, Intro to Business Final Review, INTRODUCTION TO BUSINESS FINAL EXAM

Ace your homework & exams now with Quizwiz!

Database

A single collection of data stored in one place and it's used by employees throughout the organization

One person maintaining a business is known as

Sole proprietorship

"Stakeholders" refers only to the owners of a business.

False

private investment company that is licensed by the SBA to make loans to small businesses.

A Small Business Investment Company (SBIC) is a(n):

Factor

A firm that specializes in buying other firms accounts receivable

to reduce the inequality in the distribution of wealth.

A major premise of a socialist system is:

Collection of data needed to establish an Mis results in

Databank

A disadvantage of corporations is that they generally require extensive paperwork

True Page 127

A manager with a lot of subordinates is said to have a

Wide span of control

Which of the following is part of the marketing process?

determining the brand name for a new product

A __________ is a study of what is done by employees who fill various job titles.

job analysis

Distribution industry

making a service available for the consumer or business user that needs it.

Dividends

Earning of corporation that are distributed to stockholders

Hierarchy of needs was developed by

Maslow

Conversations with small business owners before you start your own business is:

a good idea since you can learn from their experiences.

Women have

a green ceiling when faced with purchasing a franchise.

Which of the following is a characteristic of successful entrepreneurs?

action-oriented

Successful entrepreneurs are self-nurturing. This means they

are able to replenish their own enthusiasm

Corporate social responsibility describes the firm's:

concern for the welfare of society

A merger involving a trucking firm and a bakery would be an example of a:

conglomerate merger.

focus

A ________ group consists of a small group of people who meet under the direction of a discussion leader to discuss opinions about an organization, its products, or other issues. concept focus peer consumer interest

mass

A ________ marketing strategy develops products and promotions designed to please large groups of people. volume mass relationship value

compressed workweek

A __________ allows workers to work the full number of hours per week in fewer days. flextime plan compressed workweek maximum plan double shift plan

merger

A ____________ is two firms combining to form one company.

A system of Exchange in which goods and services are exchanged without the use of money is known as

A barter system

Functions of a management information system

Collecting, storing, updating and processing the data into information

Maturity Date

Date on the face of a bond telling when the bond is to be repaid

monetary policy

Efforts by the Federal Reserve Bank (the Fed) to control the money supply and interest rates are known as: fiscal policy. incomes policy monetary policy securities policy.

Board of Directors

Elected by Common stockholders

not unique to the United States.

Ethical problems and issues of social responsibility are: unique to firms involved in global trade. unique to the United States. not unique to the United States. less important now than in the past.

Intangible assets

Examples - Trademarks and Good Will

A company is interested in attracting managers who can contribute New Perspective to the managerial process. This company should concentrate on what kind of recruiting

External recruiting

Macroeconomics example:

Factors that determine how fast a nation's economy is growing

The price at which shares of an open-end mutual fund are issued and redeemed is based on the fund's _____. a. face value per share b. open-end fund c. net asset value per share d. closed-end fund

a. face value per share

The balance sheet of Ranger Inc. shows its liabilities and owners' equity as $150,000 and $75,000, respectively. In this case, Ranger's ______. a. total assets are $225,000 b. total income is $75,000 c. net income for the current year is $225,000 d. net cash flow for the current year is $75,000

a. total assets are $225,000

Small business employees:

are often more satisfied with their jobs than are their counterparts in big business.

A major advantage of S corporations is that they:

avoid the problem of double taxation associated with conventional corporations.

Data gathered for MIS comes from

Internal sources

difficult and not as clear-cut as U.S. firms would like them to be.

International issues of social responsibility and ethical behavior are: the result of greed found in capitalist countries. found primarily in countries with a low standard of living. a concern only of business professors. difficult and not as clear-cut as U.S. firms would like them to be.

The organizational functions and set of processes for creating, communicating, and delivering value to customers and for managing customer relationships in ways that benefit the organization its stakeholders

Marketing

Procter & Gamble uses what kind of production to produce household goods

Mass production

When a customer purchase a product the system immediately reflects that purchase in a central computer and calculates the new inventory level what kind of system is this

Performing statistical analysis

service

The U.S. economy is now dominated by the _________ sector. farm manufacturing service trade

flatter; wider

The ________ an organization's structure, the ________ the span of control. flatter; wider flatter; narrower taller; wider taller; flatter

standard of living

The amount of goods and services people can buy with the money they have is called their: nominal income. consumer price index. profit margin. standard of living.

Stockholders

The board of directors for a corporation is elected by its: Creditors. Stockholders. Managers. Employees.

If a good or service satisfy a human need it has

Utility

Allowing family leave and flextime

Which of the following business policies would most likely help a single-parent family? Requiring the employee to work regularly scheduled hours each day. Allowing sick leave to be used only if the employee is ill. Requiring education outside of regular business hours to help the employee advance. Allowing family leave and flextime.

With limited resources, firms can better satisfy the wants and needs of a smaller target market.

Which of the following correctly identifies the motivation of firms using a market segmentation strategy? Firms can avoid the government's anti-trust laws by selecting a small slice of a larger market. This strategy enables a firm to identify common traits in different consumer groups. With limited resources, firms can better satisfy the wants and needs of a smaller target market. Government regulations require firms to maintain accurate records to show that the firm has not discriminated against a market segment.

Cross-functional self-managed teams

Which of the following describes a group of employees from different departments working together on a long-term basis?

The right to compete

Which of the following is a basic right under capitalism? Freedom of collusion. Freedom from taxation. The right to vote. The right to compete.

knowledge

Which of the following is a factor of production? bonds Money Knowledge Service

determining an appropriate brand name for a new product

Which of the following is part of the marketing process? developing an understanding of company finances available for new products determining an appropriate brand name for a new product designing the most efficient method of production of a new product contacting suppliers after a purchase to ensure producer satisfaction

______ are the inexpensive goods and services that consumers buy frequently with limited consideration and analysis. a. Shopping products b. Convenience products c. Unsought products d. Specialty products

b. Convenience products

____ includes all business donations to nonprofit groups, including both money and products. a. Corporate compensation b. Corporate philanthropy c. Corporate remuneration d. Corporate marketing

b. Corporate philanthropy

_____ are files of related facts and figures organized according to a logical system and store on hard drives or some other computer-accessible storage media. a. Extranets b. Databases c. Intranets d. Data mines

b. Databases

Minimize taxes and regulations.

Which of the following policies would tend to foster entrepreneurship? Minimize taxes and regulations. Promote more regulations to protect the environment. Develop policies to reduce corruption between individuals. Encourage public ownership of businesses.

______ are best described as specialized Internet sites, which provide a platform to interact for businesses, specifically those involved in business-to-business (B2B) transactions. a. Social networking sites b. E-marketplaces c. Blogs d. I-commerce sites

b. E-marketplaces

laptop computers

Which of the following products would normally be classified as a shopping good or service? laptop computers newspapers fur coats chewing gum

Which of the following is a characteristic of Six Sigma? a. It exercises tight control on inventories of goods. b. It aims to reduce defects to no more than 3.4 per million opportunities. c. It focuses on the correction rather than the prevention of mistakes. d. It lays emphases on using green technologies for environmental protection.

b. It aims to reduce defects to no more than 3.4 per million opportunities.

Which of the following is true of a partnership? a. It is the most expensive form of direct investment. b. It is a formal, long-term agreement. c. It is the most basic level of global trade. d. It is otherwise known as domestic franchising.

b. It is a formal, long-term agreement.

Shared accountability among employees.

Which of the following would be a unique focus of an integrity-based ethics code?

Which of the following is a disadvantage of holding larger inventories? a. increasing ordering costs b. increased risk of stock-outs and lost sales c. additional holding costs d. irregular production schedules

c. additional holding costs

Balance of payments deficit means that: a. foreign borrowing and lending are excluded from calculating the balance of payments. b. more money flows in than out. c. more money flows out than in. d. foreign payments and receipts are excluded from calculating the balance of payments.

c. more money flows out than in.

A(n) ____ is a set of related activities that transform inputs into outputs, thus adding value. a. divestiture b. poka-yoke c. process d. event

c. process

A company that values its employees: a. does not worry about its profits as long as its employees are satisfied. b. does not delegate decision making of its employees. c. strives to recognize the untapped potential of its employees. d. allows employees to take paid vacations as many times as they want.

c. strives to recognize the untapped potential of its employees.

A(n) ___________________ is a state-chartered legal entity with authority to act and to have liability separate from its owners.

conventional corporation

they are not in agreement with what it means to be socially responsible.

1. Although college graduates would prefer to work for socially responsible companies, they would not consider less pay to do so. they are not interested in companies that spend resources are corporate social initiatives. they are not in agreement with what it means to be socially responsible. they favor Milton Friedman's philosophy on the subject.

job sharing.

1. At a large law firm, Gina works daily as a secretary from 9:00 a.m. until 1:00 p.m., when she is replaced by Cassie who works from 1:00 until 5:00 p.m. The arrangement Gina and Cassie have at the law firm is known as: job enlargement. job sharing. job rotation. job allocation.

_____ is the benefit country has in a given industry if it can make products at a lower opportunity cost than other countries. a. Absolute advantage b. Service differentiation c. Product differentiation d. Comparative advantage

d. Comparative advantage

_____ refers to planning for unexpected events, usually involving a range of scenarios and assumptions that differ from the assumptions behind the core plans. a. Strategic planning b. Tactical planning c. Operational planning d. Contingency planning

d. Contingency planning

Socially responsible employers: a. Make use of strategies such as planned obsolescence. b. Create workplace environment that only values profit. c. Provide minimal safety requirements. d. Ensure that hard work, commitment, and talent pay off.

d. Ensure that hard work, commitment, and talent pay off.

In the context of strategic planning, which of the following is true of mission statements? a. They contain complex words and phrases. b. They follow a common format across various organizations. c. They are circulated within an organization, not outside. d. They provide a clear, long-term focus for an organization.

d. They provide a clean, long-term focus for an organization.

Compared to the U.S., China and India have: a. smaller market size b. higher GDP growth rates c. higher per capita GDPs d. limited market potential

? quiz 2, answer isn't D

market

A _______________ refers to people with unsatisfied wants and needs who are willing and able to buy a product or service.

describe all aspects of the business, including product, the target market, the nature of the competition, any financial resources, and the owner's qualifications.

A good business plan will: focus on the big picture and omit the small details of operating the business. describe all aspects of the business, including product, the target market, the nature of the competition, any financial resources, and the owner's qualifications. be purposefully vague, since specifics can restrict creativity and flexibility. focus on the nature of the business and the target market, omitting personal information about the owner in order to avoid personality issues.

Assets refer exclusively to: a. the taxes to be paid by a firm b. the wages payable by a firm to its employees c. the cash payable by a firm d. the resources owned by a firm

d. the resources owned by a firm

avoid the problem of double taxation associated with conventional corporations.

A major advantage of S corporations is that they:

Emphasis on social equality

A major benefit of socialism is the: Ability to stimulate rapid economic growth. Ability to keep taxes low. Emphasis on social equality. Emphasis on private enterprise instead of public ownership.

Statistics

A measure that summarizes a particular characteristics of a group of numbers

Merchant wholesaler

A middleman that purchases Goods in large quantities then sell them to retailers

What do franchisees typically have to pay to the franchisor

A one one-time fee and the monthly royalties based on sales

whistleblower.

A phone call to a government official by an employee of the Enron Corporation disclosed the deception and dishonesty of the firm's accounting records. This employee played the role of a: whistleblower. horn blower. cheerleader. moral policeman.

A sales allowance

A price reduction offered to customers who accept damaged or soiled goods

meet with experienced entrepreneurs and learn from their advice.

A small business management course at a community college may offer entrepreneurs an opportunity to:

A relatively permanent committee charged with performing reoccurring is called

A standing committee

an explanation of why the company exists and where it wants to go.

A vision for a company is: an explanation of why the company exists and where it wants to go. the same thing as an objective. a detailed plan that outlines the major organizational structure of the firm. very relevant for the short-term, but less useful for the long-term.

A common planning Horizon for production activities is

A year

compliance

A(n) _______ -based ethics code emphasizes the prevention of unlawful behavior by increasing control and penalizing wrongdoers. legal corporate responsibility compliance integrity

union

A(n) ________ is an employee organization that represents workers in employee-management bargaining over job-related issues. trades guild union ESOP cross-functional team

organization chart

A(n) __________ is used by an organization to show who is accountable for the completion of specific work and who reports to whom. balance sheet Venn diagram stakeholder outline organization chart

primary boycott

A(n) __________ occurs when a union encourages its members and the general public not to buy the products of a firm involved in a labor dispute. injunction wildcat strike primary boycott embargo

may be classified as a small business depending upon the size of other firms in the industry.

According to SBA guidelines, a manufacturing firm with 1,000 employees: would be classified as a large firm in the industrial sector. may be classified as a small business depending upon the size of other firms in the industry. is too small to achieve the efficiencies of large scale production. is classified as an S corporation.

According to the "Dealing with Change" box in Chapter 1, which of the following statements about Amazon.com is the most accurate?

Adjustments to the market have made Amazon.com one of the fastest-growing companies in the United States.

Rewards individuals who are willing to link a company's website to their blog or social network page.

Affiliate marketing is a web-based business strategy that: relies on customers that enter the sites of large companies such as GAP™ and Nike™. asks you to offer the names and emails of ten of your friends who may be interested in the products or services the web-site is selling. is currently illegal, but difficult to stop. Rewards individuals who are willing to link a company's website to their blog or social network page.

assess future human resource demand

After preparing a job analysis, the next step in the human resource planning process is to: develop a human resource inventory. assess future human resource demand. develop a human resources budget. establish a strategic plan.

Financial Management

All activities concerned with obtaining money and using it effectively

find a need in the global market to flll it

An important principle for firms desiring to export is: sell to countries with high standards of living. keep in mind that American methods are probably always best. find a need in the global market and fill it. if your product sells well in the U.S. it will sell well globally.

direct marketing.

Any marketing method that directly links manufacturers or intermediaries with the ultimate consumer could be classified as: direct selling. online marketing. direct marketing. kiosk-based.

Advertising

Any non-personal promotional message that is paid for by a identified sponsor and it's directed towards a selected audience

a different marketing strategy.

As a product progresses through each stage of the product life cycle, successful marketers recognize that each stage requires: a different marketing strategy. a consistent application of fundamental marketing practices. an increase in the amount spent on advertising. more emphasis on personal selling.

___________ is the characteristic of work concerned with the degree of freedom, independence and discretion an employee has in scheduling work and determining procedures.

Autonomy

The reinforcement Theory says that people are motivated by

By receiving Rewards or punishments based on their behavior

faster

Compared to railroads and trucks, the biggest advantage offered by air transport is that it is: faster. more dependable. lower in cost. able to reach more destinations.

Production industry

Construction Mining Manufacturing

Consumer Product Classification

Convenience Shopping Specialty

salary pay system

Dana is a supervisor at a small manufacturing plant. Her pay starts at $635 per week. Dana is paid on a(n): salary pay system. commission pay plan. rolling rate system. income averaging plan.

Long Term Liabilities

Debts to be repaid in 2 years or less

Cars manufacturers tend to use what approach in teaching their target market

Differentiated

retail stores.

Discount stores, supermarkets, and department stores are all common types of: multilevel marketers. full-service wholesalers. retail stores. category killers.

Current ratio

Divide Crrent assets by current liabilities

A distribution of earnings to the stockholders of a corporation is a

Dividend

Behavioral segmentation.

Dividing the market based on how people behave toward various products.

In the state of which a business is incorporated this is known as a

Domestic Corporation

________ used the terms Theory X and Theory Y to describe different attitudes managers had about employees.

Douglas McGregor

Overpopulation

Economist Thomas Malthus believed that ______________ would limit economic progress. depletion of gold reserves Overpopulation the tendency of governments to levy high tax rates the inability of workers in developed nations to compete against cheap foreign labor

producing goods and services using the least amount of resources.

Efficiency means: producing the desired result. producing goods and services using the least amount of resources. the amount of output you generate given the amount of input. the buying and selling of goods using the most amount of resources.

Online Farms take a business-to-customer model approach by

Emphasizing customer focus Try to build a long-term relationship with the customers try to understand how the customer being online

physiological

Employees in the United States and other developed nations are least likely to be motivated by the desire to satisfy __________ needs: social esteem physiological self-actualization

believe the reward they receive fairly compensates them for their efforts when compared to others in similar positions.

Equity theory states that employees will perform well if they: understand the nature of their responsibilities. have an opportunity to impress others in the organization. believe the reward they receive fairly compensates them for their efforts when compared to others in similar positions. know that their supervisor has set reasonable performance goals and communicated them clearly.

A manager is an individual who assumes the risk of starting a business

False

According to the "Dealing with Change" box in Chapter 3, one out of every six people on Earth lives in either China or India.

False

Adapting to changes in demand is easy for businesses to do.

False

The SCORE (Service Corps of Retired Executives) program, sponsored by the SBA, is designed to provide financial assistance to minorities and women.

False, SCORE usually has an office with volunteers from industry, trade associations, and education who counsel at no cost (except for expenses).

espirit de corps

Fayol's ____________ principle says that workers' attitudes would create an environment of pride and loyalty within an organization. esprit de corps hierarchy of loyalty unity of command Equity

unity of command

Fayol's _____________ principle states that each worker should report to one, and only one, boss. division of labor unity of command esprit de corps comparative advantage

cash flow statement

Financial statement that provides information about a firm's operating, investing and financing activities in a specific period of time

fixed-position layout

Firms often use a(n) _____________ when working on a major project involving the production of a large item such as an airplane, ship, or bridge. assembly line layout continuous production process fixed-position layout process manufacturing technique

As long as the employee is putting a certain amount of hours at the end of the week this plan is called

Flex time

trust and cooperation.

Following an ethics-based approach to decision making will normally lead to higher: profits. trust and cooperation. prices. employee turnover.

lower employee turnover.

Following an ethics-based approach to decision making will normally lead to:

a sense of achievement.

Herzberg's research found that the factor that ranked highest as a motivator was: the friendliness of supervisors. a sense of achievement. a compensation system that included bonuses and regular pay raises. the presence of clear and consistent company rules and policies.

strike

Historically, the most potent union tactic when collective bargaining efforts break down has been the: strike. lockout. court injunction. primary boycott.

controlling

Ho Han has just spent two hours going over quality reports to determine whether his department is meeting quality standards he set for the current month. Ho's efforts are part of the ___________ function of management.

When Comcast and Time Warner Cable decided to become one company this is a example of

Horizontal takeover

Taking the firm private

If a group of stockholders or management obtain all the stock of a previously publicly traded firm for themselves, this is referred to as: Capitalizing. Stock turning. Turning the equity. Taking the firm private.

Price of the good will rise

If a shortage exists for a good in a free-market economy, the: Price of the good will rise. Price of the good will fall. Government will order suppliers to increase the production of that good. Government must establish a rationing system to make sure that the good is fairly distributed.

Increase interest rates

If businesses are producing at capacity, and the nation is experiencing almost full employment (a very low rate of unemployment - less than 2%), the Fed may decide to: Lower interest rates. Raise taxes. Lower taxes. Increase interest rates.

social needs

In Maslow's hierarchy of needs, the desire for love and acceptance would fall into the category of: social needs. esteem needs. primal needs. self-actualization needs.

administered

In a(n) __________ distribution system the producer manages all of the marketing functions at the retail level. administered contractual conglomerate master limited

How is it the people working in their own self-interest produce goods, services, and wealth for others?

In order to earn money and produce goods that improve lives, self-directed gain would provide jobs, and subsequently wages for others.

Federal Reserve Bank

In the U.S., the __________________________ has the role of managing the money supply and interest rates.

investing in companies who are committed to a better environment.

Investors, today are realizing financial benefit by: investing in companies who only operate in stable environments. investing in firms that have abundant resources for social initiatives. investing in companies who are committed to a better environment. investing in companies who are leaving a sizeable carbon footprint.

A partnership formed to operate for a specific time period or to accomplish a specific purpose is known as

Joint venture

A partnership with foreign country that can provide immediate market knowledge and access comedies risk and control over product attributes is a

Joint venture

will use the firm's existing resources to develop new products.

Ketan is an intrapreneur for a multinational electronics company. As an intrapreneur Ketan: is expected to recruit new managerial talent for his firm. will use the firm's existing resources to develop new products. is concerned mainly with improving the firm's public image. will concentrate on creating new ventures that can be spun off to become independent firms.

Line of Credit

Loan approved before the money is actually needed

convenience goods

Location is very important in the marketing of ________ because consumers desire to purchase these goods with a minimum of shopping effort. unsought goods shopping goods specialty goods convenience goods

Activities that increase employee satisfaction such a as satisfaction surveys, employee communication programs, exit interviews, and fair treatment is all apart of which phase of the human resource management process

Maintaining Human Resources

Marketing concept

Make the customers happy to make more profits

the type of manager that would be most concerned with products offered by its competitors

Marketing managers

franchise

McDonald's, KFC, Baskin-Robbins, and AAMCO all make use of the __________ form of contractual distribution system. wholesale-sponsored chain franchise retail cooperative horizontally administered

social needs

Most employees in less-developed nations are motivated by the desire to satisfy: physiological needs. social needs. esteem needs. empathetic needs.

the market potential for the product or service, at the prices you need to charge.

One important consideration when prospecting for a good franchise business is:

both goods and services

Operations management is a specialized area in management that converts resources into: services, rather than goods. goods, rather than services. both goods and services. financial data.

In line with a franchising agreement the franchisee may use

Patents design business know-how Geographic exclusivity Trademark and copyright

Is a firm post employee openings along with the required skills that is needed on its website it is using the internet to

Recruit

A company is interested in verifying previous job responsibilities held by its applicants. The selection technique that is most helpful in obtaining this information is the

Reference check

Warehousing

Refers to the set of activities involved with receiving and storing goods and preparing them for reshipment

Which of the following stages are not a part of Team development process

Reminding The team development process includes Forming Storming Norming Performing Adjoining

__________ provided dynamic leadership for the American Federation of Labor during its early years.

Samuel Gompers

Unsecured Financing

Short term financing not backed by collateral

60-80

Small businesses create about __________ percent of the new jobs in the United States. 5-10 25-35 60-80 100

can be offered in creative ways that enable the firm to offer an attractive benefit to workers with young children.

Small businesses have found that child care programs: are too expensive to offer, which puts them at a serious disadvantage when competing against larger firms to attract the best workers. can be offered in creative ways that enable the firm to offer an attractive benefit to workers with young children. are less necessary now than they were before the federal government expanded its child care programs. are not very expensive, but tend to be more trouble than they are worth.

individualized customer service as opposed to impersonal corporate bureaucracies.

Small businesses often have advantages in international markets. One advantage is:

support and assist the line positions.

Staff positions in an organization: support and assist the line positions. are superior to line positions. are less well-paid than line positions. are ways to employ past CEOs until they retire.

shares of ownership of a corporation is known as

Stock

bring a varity of talent to the business venture

Successful entrepreneurial teams: quickly organize themselves like large corporations. assign only one person to new product innovation. bring a variety of talent to the business venture. outsource most of the details of innovation.

formal organization

The ____________ of a business firm is the system that details lines of authority, responsibility, and position, similar to the structure on organization charts.

production era

The business philosophy of "produce as much as you can because there is a limitless market" is consistent with which of the following marketing eras? production era selling era marketing era customer relationship era

eliminated the jobs of some

The downsizing that has occurred in recent years has __________ middle managers in many organizations. increased the number of eliminated the jobs of some strengthened the control of increased the importance of

communism

The economic and political system in which the government owns almost all the major factors of production is called _______________________________

Y

The generation born between 1980 and 2000 is known as Generation: We Me two Y Z

The more information a manager has....

The less risk when making a decision

information-based sector

The new era of business is likely to be dominated by the: service sector. information-based sector. manufacturing sector. continued decline in productivity.

departmentalization

The process of setting up individual functional units of the business to do specialized tasks is called:

Give members more economic power as a group than they would have as individuals.

The purpose of a farm cooperative is to: Give members more economic power as a group than they would have as individuals. Give each farm an equal share in the running of the cooperative. Equalize the members' standard of living. Allow socialism a foothold in the U.S.

facilitate global trade

The purpose of the General Agreement on Tariffs and Trade (GATT) is to: facilitate global trade. create common market arrangements in specific geographical regions. negotiate an end to civil wars. establish a set of international property right laws.

factors of production

The resources that contribute to the creation of wealth are known as ________. production coefficients factors of production production technologies production aggregates

Basic right under capitalism:

The right to compete

met the needs of our nation ewhen it was conceived, but needs an overhaul.

The social security system, continues to burden businesses due to the fact that businesses must provide and pay for this benefit for their employees. met the needs of our nation ewhen it was conceived, but needs an overhaul. will continue to serve the needs of U.S. citizens into the distant future, due to the fact that there are so few retirees placing a strain on it. is failing because the U.S. government failed to invest the money that employees contributed.

function.

The traditional technique used to departmentalize an organization is by:

one or two suppliers because firms share so much information.

Today manufacturers are relying more heavily on: one or two suppliers because firms share so much information. the bidding process to obtain the lowest price. many suppliers to keep its leverage. developing an MRP system for purchasing.

respond readily to customer demands.

Today's preference to flat vs. tall organization structures is mainly due to a flat organization's ability to: encourage esprit de corps among employees in various departments. increase the cost of labor through economies of scale. respond readily to customer demands. increase sales through multiple customer contacts by several layers of management.

A closed corporation is one whose stock is held by a few people and isn't available to the general public.

True Page: 126, figure 5.3

A cooperative consists of people with similar needs who pool their resources for mutual gain.

True Page: 141

A business should be managed ethically to maintain a good reputation.

True Page 94

The __________ on packages were intended to improve inventory control for businesses.

Universal Product Codes

agents and brokers

Unlike merchant wholesalers, ___________ never actually own the goods they help to distribute. rack jobbers retailers cash-and-carry wholesalers agents and brokers

when you own your own business you are responsible for all the business debts.

Unlimited liability means:

Equity Capital

Used to start a business regardless of its legal form

corporate philanthropy

We describe charitable donations by corporations to nonprofit organizations as _________. corporate philanthropy corporate responsibility corporate strategy structural commitment

Making donations that are directly related to a firm's corporate competency.

When a firm undertakes corporate social initiatives it is:

Making donations that are directly related to a firm's corporate competency.

When a firm undertakes corporate social initiatives it is: Making donations that are directly related to a firm's corporate competency. Making long-term commitments to one cause. Hiring minority workers. Making safe products.

The sole proprietor's heirs have the option of taking over the business.

When a sole proprietor dies: The sole proprietor's heirs have the option of taking over the business. The business is sold to a larger corporation. The company continues to function as it always has. The company always closes down.

small firms often can offer more personalized customer services than big firms.

When comparing small businesses to their larger competitors, it is clear that: bigger is always better. small firms can always outperform big firms on a level playing field. small firms often can offer more personalized customer services than big firms. small firms are slower to respond to opportunities than are large firms.

each of us

When discussing moral and ethical issues, it is helpful to remember that ethical behavior begins with ___________________.

planning

When managers identify a market trend that suggests a new opportunity and then devise a strategy to go after this new opportunity, they are involved in the function of:

Giving front-line workers additional resources needed to provide faster and better service to customers

Which of the following activities is part of the organizing function of management? Going over a spreadsheet to analyze some financial data Teaching a new employee how to use a piece of office equipment Giving front-line workers additional resources needed to provide faster and better service to customers Developing a broad general strategy to help the organization meet its long-term growth objectives

Cross-functional self-managed teams

Which of the following describes a group of employees from different departments working together on a long-term basis? Focus group Mixed function groups Cross-functional self-managed teams Intrapraneurial teams

a consumer orientation

Which of the following is a part of the marketing concept? a consumer orientation avoidance of risk financial return orientation emphasis on efficient production

Limited liability of owners

Which of the following is an advantage of the corporate form of business when compared to sole proprietorships and partnerships?

Religion

Which of the following sociocultural forces can have a significant impact on global business operations? Religion Universal sales standards Single-phase marketing Global marketing

A major drawback of a partnership is that it is difficult to terminate.

Which of the following statements about partnerships is most accurate?

A small business can maneuver and change quicker than most large businesses can. It can react to market conditions and customer concerns much faster.

Which of the following statements accurately depicts an advantage that small business has over big business?

Some highly visible strikes in recent years show that the strike is not dead as a labor tactic, but very few labor disputes actually lead to a strike.

Which of the following statements best summarizes the extent to which unions currently rely on strikes as a tactic when collective bargaining breaks down? Unions have become increasingly reliant on strikes as other, less confrontational, tactics have proven unsuccessful. Some highly visible strikes in recent years show that the strike is not dead as a labor tactic, but very few labor disputes actually lead to a strike. Unions have almost completely avoided strikes since President Reagan replaced striking air traffic controllers with strikebreakers in 1981. Strikes organized by unions are now less common than lockouts by management, indicating a major shift in attitudes between labor and management.

Most firms are finding that the cultural changes required by ADA are more difficult to implement than the structural changes

Which of the following statements is the most accurate description of the impact the Americans with Disabilities Act of 1990 (ADA) has had on businesses? ADA has had no real impact, since most businesses have always been sensitive to the needs of disabled workers. Most firms have found that the structural changes needed to comply with ADA requirements have been very difficult and expensive to implement. Most firms are finding that the cultural changes required by ADA are more difficult to implement than the structural changes. Businesses have found that even with the changes required by ADA, disabled workers are simply not capable of handling most types of jobs.

Are taxed at the owner's personal tax rate

Which of the following statements is the most accurate? Sole proprietorships: Are well suited for people who want to own a business and share in its profits without taking an active role in management. Are taxed at the owner's personal tax rate. Is the least risky form of business ownership. Must receive a state charter before they can legally conduct business.

evaluating various distribution alternatives to determine the best way to get the product to the consumer

Which of the following tasks would be included in the marketing mix? evaluating various distribution alternatives to determine the best way to get the product to the consumer examining various sources of funds to determine the best way to finance an expansion of the organization's production facilities assigning tasks to specific workers on specialty product assembly lines developing financial statements based on the sales transactions of the firm during a specific time period

an accountant hired to help during the busy tax season

Which of the following workers would be the best example of a contingent worker? an assembly line worker who belongs to a union and has 17 years of seniority an accountant hired to help during the busy tax season an elementary school teacher who just received tenure an electrical engineer who frequently travels out of the country to the various manufacturing facilities owned and operated by his firm

Factors that determine how fast a nation's economy is growing.

Which of the following would be a topic emphasized in a macroeconomics course? Factors that determine how fast a nation's economy is growing. How a firm decides the amount it will charge for one of its products. How a consumer chooses which goods to buy. The determinants of the wage rate for labor in the auto industry.

certain work related factors satisfied higher order needs, while other work related factors satisfied lower order needs.

While Maslow's research categorized a person's needs in terms of higher order and lower order needs, Herzberg's research recognized that: employees aspire to satisfy lower order needs at work and higher order needs through other life activities. certain work related factors satisfied higher order needs, while other work related factors satisfied lower order needs. the physical environment and general working conditions were of no consequence to employees. These factors did not affect higher or lower order needs. hygiene factors satisfied higher order needs and motivating factors satisfied lower order needs.

Which of the following correctly identifies the motivation of firms using a market segmentation strategy?

With limited resources, firms can better satisfy the wants and needs of a smaller target market.

Offering _________ can be a useful way for a firm to downsize without causing a serious decline in the morale of the remaining employees.

early retirement

Which business function involves credit management/collecting funds from customers?

finance

One way a store selling big screen televisions might add ________ utility is to provide convenient delivery and set up to customers.

possession

The first step in the human resources planning process is to:

prepare a human resource inventory.

After earning $30 million in net income, Rolatrim Industries distributed $5 million in dividends to their stockholders. The board of directors of the firm decided to invest the remaining $25 million back into the business. This $25 million reinvestment of profits represents:

retained earnings.

A _________ forecast predicts the revenues, costs, and expenses a firm will incur for a period of one year or less.

short-term

In coming years, high tech growth areas such as computers, biotechnology, and robots are likely to experience a:

shortage of skilled labor.

The degree to which a job has a substantial impact on the lives or work of others in the organization is referred to as:

task significance

Brainstorming

__________ is a problem solving technique that attempts to come up with as many solutions as possible in a short time without censoring the ideas. Brainstorming Game theory CAD/CAM PMI

Fiscal policy

____________ refers to the efforts of the federal government to keep the economy stable by increasing or decreasing taxes and/or government spending.

Insourcing

_____________ refers to companies headquartered abroad who elect to bring production and manufacturing to domestic subsidiaries. Outsourcing Insourcing Production transition Production transformation

Piggyback

_____________ shipping is the term used when truck trailers are transported on rail flatcars. Trailerhitch Piggyback Dualmode Trailerail

Bureaucratic

______________ organizations are characteristic of many layers of management and function with well-defined rules and procedures. Matrix Inverted Ordered Bureaucratic

Objectives

__________________ are specific, short-term, measurable results an organization wants to achieve in order to fulfill its long-term goals.

Hansel owns and runs a fruit juice manufacturing company. He decides that the company will sell packaged fruit juices through wholesalers and retailers. In this scenario, which of the following distribution strategies is Hansel planning to use? a. Channel intermediaries b. Direct channel c. Voicemail marketing d. Direct mail

a. Channel intermediaries

U.S. corporations are forbidden to offer bribes since 1977 under the _____. a. Foreign Corrupt Practices Act b. Sarbanes Oxley Act c. Foreign Enlistment Act d. Corruption of Foreign Public Officials Act

a. Foreign Corrupt Practices Act

Which of the following is the most costly form of foreign direct investment? a. Offshoring b. Franchising c. Importing d. Licensing

a. Offshoring

Which of the following marketing strategies can be best employed for attracting tourists? a. Place marketing b. Idea marketing c. Green marketing d. People marketing

a. Place marketing

Local offices affiliated with the Small Business Administration (SBA) that provide comprehensive management assistance to current and prospective small business owners are known as _______. a. Small Business Development Centers b. Small Business Incubation Offices c. Small Business Advancement Stations d. Small Business Creation Sites

a. Small Business Development Centers

_____ is the rate which a new product moves from conception to commercialization. a. Speed-to-market b. Picketing c. Hard sell d. Leveraging

a. Speed-to-market

______ refers to the quantity of products that producers are willing to offer for sale at different market prices . a. Supply b. Demand c. Ration d. Rebate

a. Supply

A(n) _____ is similar to a virus, except that it is an independent program that can be spread across computer networks without being attached to other files. a. worm b. spam program c. spyware d. adware

a. Worm

Steaks & Grillz, a restaurant, uses pleasant music and lighting along with mpressive decor to attract customers. In this scenario, the restaurant is attempting to enhance its _____. a. ambience b. signage c. dissonance d. functionality

a. ambience

Pyoiunalek's, a U.S. restaurant chain, markets and endorses its business in Germany by offering buyers the rights to launch and operate the restaurant in their own country. It lends financial and marketing assistance to the buyers. Pyoiunalek's is employing the strategy of _____. a. foreign franchising b. personal selling c. joint venture d. importing

a. foreign franchising

Nursing jobs usually have a high level of _____ due to the fact that the work performed by nurses has a large and direct impact on the welfare of others. a. task significance b. task identity c. skill variety d. strategic planning

a. task significance

The physical good or delivered service that provides a core benefit is called the ______ product. a. Pure b. Actual c. Secondary d. Augmented

b. Actual

In the context of factors of production, which of the following is an example of a natural resource? a. An equipment b. Land c. An investment d. Information

b. Land

which of the following bodies makes U.S. accounting practices more consistent with those in other nations along with the Financial Accounting Standards Board (FASB)? a. The Financial Accounting Foundation (FAF) b. The International Accounting Standards Board (IASB) c. The U.S. Federal Reserve (Fed) d. The Securities and Exchange Commission (SEC)

b. The International Accounting Standards Board (IASB)

Which of the following is a disadvantage of foreign outsourcing? a. The cost of shipping goods can be very high. b. The risk of poor quality control becomes higher. c. The cost of production becomes irrecoverable. d. The foreign company pays higher wages to the workers.

b. The risk of poor quality control becomes higher.

Juan, the CEO of a technological firm, believes that the firm's employees are intelligent, creative, and hardworking. He is convinced that they are passionate about their work, and they can be further motivated through rewards. In this scenario, Juan's views about his workers is consistent with _____ assumptions. a. Theory X b. Theory Y c. hierarchy of needs theory d. equity theory

b. Theory Y

Identify a true statement about nonprofit organizations. a. They are usually in the business of financial gain. b. They employ people and take in revenue. c. They do not produce goods and services. d. They are not business-like establishments.

b. They employ people and take in revenue.

Which of the following pieces of information is found in the Small Business Administration (SBA) resources? a. health tips for entrepreneurs b. industry-specific statistics c. updates on multibillion-dollar business regulations d. tips on making a company public

b. industry-specific statistics

Which of the following is the scope of relationship of marketers who actively gather data and pursue a connect with customers who initiate contact? a. partial partnership b. limited relationship c. full partnership d. valued relationship

b. limited relationship

For financial managers to be socially responsible, it is necessary that they: a. focus solely on the short-term performance of the firm b. make a long-term commitment to the needs of stakeholders c. focus on maximum utilization of human resources d. allot 10 percent of the company profit to employee welfare

b. make a long-term commitment to the needs of stakeholders

The spectrum of socially responsible behavior of firms may vary from: a. no profits to generous profits b. no contributions to proactive contribution c. intangible ideas to tangible ideas d. business initiatives to proactive business dealings

b. no contributions to proactive contribution

Which of the following refers to the claims that stockholders have against their firm's assets? a. loans payable b. owners' equity c. liabilities d. explicit costs

b. owners' equity

The characteristics of a product that a marketer offers are known as _____. a. product benefits b. product features c. product qualities d. product visions

b. product features

Through the generally accepted accounting principles (GAAP), the Financial Accounting Standards Board (FASB) aims to ensure that financial statements are ______. a. comprehensive, exhaustive, extensive, and independent b. relevant, reliable, consistent, and comparable c. cursory, intensive, unabridged, and superficial d. circumstantial, inferential, presumptive, and provisional

b. relevant, reliable, consistent, and comparable

Neon Bolt is an energy drink manufacturer. The marketing strategies of Neon Bolt are focused on males in the age group of 16-25 years who enjoy sports and other outdoor activities. The company advertises its product at skateboard parks, basketball venues, and ski resorts. The customer group that Neon Bolt focuses on is called its _____. a. marketing mix b. target market c. niche market d. market share

b. target market

According to Maslow, a higher order need:

becomes a source of motivation after lower order needs are satisfied.

When people from various countries and educational and work experience background come together to work on a project they often have varying perspective on the given issues. This benefits the company in the form of better problem solving

better problem solving

Horizontal Merger

between firms that make and sell similar products/services in similar markets

Vertical Merger

between firms that operate at different levels in the production and marketing of a product

Apple Computer's "smart team" is an example of a firm that succeeded by:

building a group of experienced managers and emphasizing teamwork and cooperation to conceive, develop, and market products.

______ software takes the electronic design for a product and creates the programmed instructions that robots must follow to produce that product as efficiently as possible. a. Computer-aided design b. Enterprise-resource planning c. Computer-aided manufacturing d. Just-in-time production

c. Computer-aided manufacturing

In the context of barriers to international trade, which of the following is an example of energy infrastructure in a country? a. Railroads b. Cell phone coverage c. Power plants d. Radio

c. Power plants

A management consulting firm offers a management training program that instructs students on utilizing various tools for effective time management and aims to improve their risk management skills. Which form of product is the firm offering? a. Pure goods b. Partial goods c. Pure services d. Partial services

c. Pure services

_____ refers to specific, shorter-term planning that applies strategic plans to specific functional areas. a. Contingency planning b. Operational planning c. Tactical planning d. Stategic planning

c. Tactical planning

Which of the following bodies assists entrepreneurs by guaranteeing loans from local commercial lenders? a. The Federal Reserve b. The Patent and Trademark Office c. The Small Business Administration d. The Department of the Treasury

c. The Small Business Administration

Which of the following statements is true of the members of a distribution channel? a. All channels have at least five channel intermediaries b. A producer follows a single channel of distribution c. The businesses that buy from wholesalers can be retailers, other wholesalers, or business users. d. Retailers buy products from the producer and sell them to businesses or other nonfinal users

c. The businesses that buy from wholesalers can be retailers, other wholesalers, or business users.

Which of the following is most likely to be true if a company adopts external recruitment methods? a. The external recruitment methods will boost the morale of the current employees. b. The external recruitment methods will lower the costs of both recruitment and training c. The external recruitment methods will tap into a range of different resources. d. The external recruitment methods will lead to a lack of diversity in the long run.

c. The external recruitment methods will tap into a range of different resources.

Habib is a writer. He regularly uses Microsoft Word for his work so he can write, edit, and prepare his manuscripts for publication. In this scenario, which of the following computer products is Habib using? a. a firewall b. antivirus software c. applications software d. operating system software

c. applications software

Data become ____ when they are processes, organized, and presented in a way that makes them useful to a decision maker in a company. a. knowledge b. values c. information d. metadata

c. information

_______ is an approach to production that emphasizes the elimination of waste in all aspects of production processes. a. The critical path method b. Enterprise resource planning c. Divesting d. Lean production

d. Lean production

When designing a retail facility, operations managers should be mindful of the _____, which is an environment where consumers and service providers interact. a. process layout b. facility location c. landscape d. servicescape

d. servicescape

Ethics refers to: a. the obligation of a business to contribute to society. b. legal codes written in the constitution. c. social movements that focus on key consumer rights. d. set of beliefs about right and wrong

d. set of beliefs about right and wrong

The interest paid on ________ financing represents a tax deductible business expense.

debt

A(n) _____________ is someone who has a good idea for a business and who sells the right to use the business name and to sell a product or service within a given territory.

franchisor

Which of the following is an advantage of the corporate form of business when compared to sole proprietorships and partnerships?

limited liability of owners

Which of the following are considered when calculating a country's balance of payments?

military expenditures

Which of the following is one of the fastest-growing segments of the U.S. economy in the last decade?

minority-owned businesses

If total liabilities were subtracted from total assets, the residual value remaining would be:

owners' equity.

Use the fundamental accounting equation to solve the following: Assets minus liabilities equals:

owners' equity.

Psychographic segmentation.

segmenting markets on the basis of personality, motives, lifestyles

The __________ is usually the easiest form of business to start and end.

sole proprietorship

Two of the most important tactics used by unions when collective bargaining efforts break down are:

strikes and boycotts.

Small business owners interested in finding out more about exporting will find that:

there is an abundance of inexpensive information from a variety of sources.

The three major types of corporate mergers are known as:

vertical, horizontal, and conglomerate.

emphasize teamwork.

The revolution in management that is currently under way suggests that the most effective managers of the future will:

A general partner can take an active role in the management of the business.

True Page: 121

"Quality is job #1" as a corporate mission statement can influence the ethical behavior of employees.

True Page 94

choose between equally unsatisfactory alternatives.

1. An ethical dilemma refers to a situation where you must: treat someone unfairly. choose between equally unsatisfactory alternatives. choose between a correct and an incorrect option. balance the concerns of profit and legality.

interfirm process

1. Operations management becomes a(n) ____________ when companies outsource several processes and create a network of firms that cooperatively produce products and services. open operation closed operation interfirm process conjoined process

The right to a discount

1. Which of the following is not a basic right proposed by former President John F. Kennedy? The right to be heard The right to be informed The right to a discount The right to safety

About __________ of the cost of things we buy are marketing costs that go to pay for distribution costs of intermediaries.

75 percent

Identify a true statement about e-commerce. a. Business-to-consumer selling companies have the larger share of profits arising from the total e-commerce sales. b. E-commerce has completely eliminated the individualized buying experience c. Business-to-business selling companies comprise the vast majority of total e-commerce sales. d. E-commerce excludes the sales of goods and services through websites or online portals.

? quiz 1

Which of the following is a true statement about profit in a business? a. It is calculated by subtracting the cost of goods from the sales revenues of a business. b. It is the financial reward that comes for starting and running a business. c. It is the goodwill that an organization gains while running a business. d. It is earned by every business all the time.

? quiz 1, answer is not A

Which of the following is a specialized type of licensing? a. Partnership b. Foreign franchising c. Joint venture d. Exporting

? quiz 2

SeeStreet is a company that produces gourmet coffee. Its employees, coffee bean growers who supply raw materials to the company, people who drink the coffee, and the people who live in the community where the company produces the coffee are called SeeStreet's _____. a. benefactors b. self-dealers c. philanthropists d. stakeholders

? quiz 3, answer is not A

vision

A __________ is an overall explanation of why an organization exists and where it is trying to head.

loss

A business incurs a ___________ if its costs and expenses exceed its revenues. loss liability debit dividend

top management

A chief operating officer (COO) would be classified as part of a firm's: primary management. middle management. top management. first-line management.

Initiating is social responsibility program takes

A commitment of employees

executives in the U.S. are compensated at a much higher rate.

A comparison of compensation of CEOs in the U.S. with compensation of top executives in Canada and Europe indicates that: U.S. CEOs are working for much lower compensation. pay for CEOs at major corporations is about the same in all of these nations. executives in the U.S. are compensated at a much higher rate. U.S. executives are paid better than average when their firms are successful, but worse than average when their firms struggle.

price leadership

A competition-based pricing strategy called __________ involves one or more dominant firms establishing the pricing practices that all competitors in an industry follow. skimming penetration target costing price leadership

Diversify business operations and investments

A conglomerate merger will: Diversify business operations and investments. Allow the firm to have a less dominant position in its market. Enable the firm to enjoy a higher degree of specialization. Give the firm a more secure access to needed materials and components and better control over quality.

reference group

A consumer's decision-making process is influenced by his/her _________, consisting of those people who help shape the consumer's beliefs, attitudes, values or behavior. reference group focus group target audience social class

Securities Exchange

A marketplace where member brokers meet to but and sell securities

the value of imports is less than the value of exports.

A favorable balance of trade occurs when the value of: imports equal the value of exports. the cash inflows equal the value of the cash outflows. the value of imports is less than the value of exports. the value of the dollar is greater than the value of the Euro.

primary

A firm gathers ___________ data by conducting surveys, interviewing customers, or mailing out questionnaires. primary secondary non-referenced unbiased

psychographic

A firm that uses __________ segmentation divides a market into groups based on values, attitudes, or interests. demographic benefit psychographic normative

make sure products meet quality standards all along the production process.

A firm would be most likely to employ statistical process control (SPC) if it wanted to: simplify the quality control process by taking samples at the end of the production process rather than examining every unit of output produced. eliminate the expenses incurred by the Deming Cycle. redefine quality control on the basis of customer needs. make sure products meet quality standards all along the production process.

the amount of carbon it releases in its effort to do business.

A firm's carbon footprint is:

strategies regarding product, price, place, and promotion.

A firm's marketing mix refers to the combination of: goods the firm offers to different market segments. advertising media the firm utilizes to promote its products. strategies regarding product, price, place, and promotion. people directly involved in making marketing decisions.

As a corporation, partnership or sole proprietorship.

A franchise can be formed: Only as a general partnership. Only as a corporation. As either a corporation or partnership, but not as a sole proprietorship. As a corporation, partnership or sole proprietorship.

The market

A free market is one in which decisions about what to produce and in what quantities are made by ____________

Agent

A functional middle man that does not take title to product and who is Complicated by commissions paid by producers

a written agreement will help reduce misunderstandings and disagreements among the partners

A good reason why partners should spell out the details of their partnership arrangements in writing is:

the firm's labor force is technically up-to-date and well trained.

A human resource inventory is designed to reveal whether: all workers are paid a fair wage. the firm has enough land and capital to use with its labor. the firm's labor force is technically up-to-date and well trained. worker morale is high or low.

the producer, but not its suppliers

A just-in-time inventory system usually reduces costs for: both the producer and its suppliers. suppliers, but not the producer. the producer, but not its suppliers. neither the supplier nor the producer, though it does lead to more flexibility for both.

managers can use both rewards and punishment to influence employee behavior

A key idea in reinforcement theory is that: goals should be set by everyone in an organization. managers can use both rewards and punishment to influence employee behavior. managers should use the same reinforcement techniques for all employees. employees are more likely to obey written instructions than verbal instructions.

A lack of cultural understanding

A sociological challenge that must be overcome in order to be successful in the global market is: A lack of cultural understanding. An overly anxious desire to please at all cost. Inequality of global natural resources. Wealth management.

A market is a group of individuals or organization that have needs for products in a given category. They have the __________, ________, and ________ to purchase products

Ability Willingness Authority

everyone with whom the business has dealings.

After developing a code of ethics, it should be communicated to:

It decreases important social interaction between and among employees on a number of levels, resulting in a change in the social network and corporate culture.

Although there are advantages and disadvantages to home-based workers for both the employer and employee the disadvantages to employer, employee, and society in general can be summarized as: It increases the overall costs to everyone: employers, employees, and society. It increases theft because employees are likely to take more office supplies home, which changes their overall attitude about honesty. It decreases important social interaction between and among employees on a number of levels, resulting in a change in the social network and corporate culture. It decreases everyone's competitive advantage.

business

An activity which seeks to earn a profit by providing a good or service is known as a(n): industry. corporation. business. service.

leveraged buyout.

An attempt by employees, management, or a group of investors to purchase an organization primarily through borrowing is called a(n):

Stockholder

An owner of a corporation is known as a(n): General partner. Limited partner. Director. Stockholder.

Trading bloc

Another name for a common market is: FTA. Trading bloc. Continental community. Economic cooperative.

the executive summary.

Bankers and potential investors are likely to read ___________ first.

develop contingency plans.

Because economic and competitive environments frequently change, organizations should:

Two firms offer the same products and needs about the same type of employees

Business model

National Labor Relations Board

Bob believes that management is treating him unfairly because of his efforts to organize a vote for union representation. Which organization should Bob contact to report his concerns? Federal Board of Labor Rights National Labor Relations Board Federal Trade Commission Federal Commission on Unfair Labor Practices

minimal government spending because it keeps taxes to a minimum.

Businesses favor: minimal government spending because it keeps taxes to a minimum. more government services because it minimizes tax consequences. maximum government spending because it raises business profits. less government spending because it lowers business profits.

Shopping

Buyers willing to expend considerable effort planning/making purchase

conform to laws and regulations.

Compliance-based ethics codes:

Markets on eBay classified as either

Consumer-to-business business-to-business

Convertible bond

Corporate bond exchanged for shares o Common stockf

concern for the welfare of society.

Corporate social responsibility describes the firm's

strikebreakers

Despite a 1938 Supreme Court ruling allowing their use, ___________ were seldom used by management during labor-management disputes until the 1980s. injunctions secondary boycotts strikebreakers yellow-dog contracts

Advantages of Sole Proprietorships

Ease of Formation Retention of Control Pride of Ownership Retention of Profits Possible Tax Advantages

Liquidity

Ease of which assets can be converted into cash

Too many people would result in food shortages and shortages of other resources.

Economics was known as the "Dismal Science" because: Too many people would result in food shortages and shortages of other resources. It was a very complex area of study based on mathematics. The world population was growing slower than expected in the 1700s, leading economists to call it a dismal period of time. There were too many old people and not enough young people to care for them.

expect the entrepreneur to provide a business plan.

Entrepreneurs applying for a bank loan should realize that bankers will:

accepting the risk of starting and running a business.

Entrepreneurship is:

Disadvantages of Corporations

Expense/complexity of formation and operation Double Taxation Paperwork and Regulation Conflicts of Interest

A foreign corporation is chartered in a country outside the U.S.

False Page: 126, figure 5.3

Opportunities for both large and small franchises.

Global franchising offers: Few opportunities for American investors. Opportunities for large franchise systems, but not small ones. Opportunities for both large and small franchises. American firms the opportunity to market goods overseas without any need to adjust for cultural differences.

Production management has been the traditional term used to describe all the activities managers do to help their firms create:

Goods

organization theory

Henri Fayol and Max Weber are best known for their contributions to: production theory. marketing theory. organization theory. finance theory.

A trade restriction that limits the amount of a particular good that may have been imported into a country during a given period of time is

Import quota

assembly-line layout

In manufacturing, a __________ is used to produce large quantities of a few types of products. assembly-line layout process layout modular layout fixed-position layout

an asset that promotes harmony among workers.

In open organizations the informal organization can be:

distance learning

Internet-based training courses are sometimes also called: contingency training. synchronized training. distance learning. vestibule training.

The most used technique of selecting a applicant

Interview

Paul wanted to know about the skills and experience needed to qualify for a management position at JCPenney. He called and asked the Personnel specialist to send him a

Job description

When sales drop each summer, the owners temporarily have too many employees to service the reduced amount of tourists. In this situation what is most likely to happen in this situation?

Layoffs workers

human relations

Leadership, coaching, and morale building are all part of a manager's _________ skills.

human relations

Leadership, coaching, and morale building are all part of a manager's _________ skills. technical theoretical empirical human relations

Prime interest rate

Lowest interest rate charge by a bank for a short term loan

A computer-based system for continually Gathering internal and external information is called

Marketing information system

The purchase of one Corporation buy another is known as

Merger

developing tactical plans and controlling.

Middle managers are responsible for: establishing the vision for the organization. assigning specific jobs and evaluating the daily performance of workers. devising the organization's strategic plan. developing tactical plans and controlling.

Capital debt

Money obtained when loans are called in

An employee's attitude towards his or her job, supervisors, and the firm itself

Morale

One product that is distributed four different uses is known to having what product mix

Narrow product mix

A manager with few subordinates is said to have a

Narrow span of control

given firms even greater flexibility in their location decisions.

New developments in information technology have: had little effect on business location decisions. given firms even greater flexibility in their location decisions. caused a movement of firms back to urban areas where access to this technology is greatest. reduced the impact government policies have on location decisions.

Employee benefits

Non-monetary Rewards

inventory turnover

Number of times a firm sells and replaces its merchandise in 1 year

limited liability partnership

One way to eliminate some of the risk of your partners making costly mistakes that could jeopardize your personal assets is to set up a:

petroleum and petroleum products.

Pipelines are used primarily to transport: coal slurry. hazardous wastes. ammonia and acids. petroleum and petroleum products.

Two important components to schedule is

Place and time

The process of shipping one thing to one place to another place is called the what utility

Place utility

Advantages of

Pooled Financial Resources Shared Responsibilities Ease of Formation Tax Advantages

Specialty

Possesses one or more unique characteristics; significant group of buyers willing to expend considerable purchasing effort

A source of Revenue flown into the farm

Revenue stream

The elements of the marketing mix are combined to

Satisfy customers

__________ consists of information that has already been researched by others and is published in journals or books, or has been made available online.

Secondary data

Gathering information about applicants for a position and choosing the most appropriate applicant is called

Selection

before a product is developed or store is opened

Similar to how large businesses proceed, small businesses make marketing decisions _____________. after the product or service is developed before the business is funded at the same time that they hire employees, research for an accountant, and prepare the business plan before a product is developed or store is opened

computer-integrated manufacturing.

Sophisticated software that enables computer-aided design machines to communicate with computer-aided manufacturing machines is called: a flexible system. a computer-based feedback control system. computer-integrated manufacturing. heuristic manufacturing integration.

valuable sources for information since you can learn from their experiences.

Successful small business owners are:

When a company purchase stock a price that is just high enough to tempt the current stock holder this action is called a

Tender offer

__________ has transformed the purchasing function in recent years by helping firms find the best suppliers and the best prices.

The Internet

autocratic

The __________ style of leadership is characterized by making managerial decisions without consulting others. autocratic bureaucratic free rein democratic

employee ideas

The greatest source of ideas for new products is: consumer suggestions. suggestions from competitors. employee ideas. research and development.

schools, hospitals and other services type industries

The immigrant population (both legal and illegal) is having a dramatic effect on car and truck makers mining and minerals industry schools, hospitals and other services type industries ice cream and confection industry

The United States

The largest importer in the global market today is: Japan. Russia. Germany. The United States.

a failure of leadership to establish ethical standards.

The majority of CEOs blame unethical employee conduct on:

Airplanes

The mode of transportation that carriers a smaller percentage of all Intercity Freight

span of control.

The optimum number of subordinates a manager can supervise is referred to as the:

span of control

The optimum number of subordinates a manager can supervise is referred to as the: manager-subordinate ratio. chain of command. employee quota. span of control.

stockholders to elect the Board of Directors.

The organizational structure of a corporation permits:

Small businesses are typically managed by

The people who start them

depends on a number of factors, and can vary from one manager to another.

The span of control for a manager: should never exceed 7 subordinates. is no longer a relevant issue in today's computerized offices. depends on a number of factors, and can vary from one manager to another. varies somewhat from manager to manager, but most managers operate best with a span of control of about 9 employees.

their income and the prices of the goods and services they buy.

The standard of living for a nation's people depends mainly on: the quality of the natural environment. how much political freedom they have. their income and the prices of the goods and services they buy. the amount of labor and capital available employed in the manufacturing sector.

The requirements to be a successful franchisee

The willingness to work long hours Willingness to face personal sacrifices Be an organized person

Douglas McGregor described two very different sets of managerial attitudes about employees, which he called

Theory X and Theory Y

By selling Christmas trees from late November through December this company is providing what utility

Time utility

Net Worth

Total assets subtract liability

A company has its own trucks that ship merchandise to its Warehouse from the supplier for the back stock purposes. What function is this

Transporting and storing

A firm desiring to enter a foreign market with a limited investment should consider foreign licensing.

True

In order to best handle a grape vine managers should

Try to eliminate it

Preferred and common

Two types of stock a company can sell

Merchandise inventory

Value of Goods on hand for sale to customers

A diagram that represents several items of information in a manner that compare is called

Visual display

may evaluate and compare a variety of factors.

When people consider purchasing a good, they: are responding to the market forces of supply and demand. are most influenced by the price of the product or service. will not consider goods that require them to make a special effort to purchase. may evaluate and compare a variety of factors.

total national debt is likely to increase.

While scanning the movies section in your local online newspaper, you click on the business section by mistake. While you're there, you quickly scan a report that says government spending will again exceed tax revenues in the current year. This means that the ______________________________________

digital natives

Young people today are often called __________ because they have never known life without the Internet.

Management by objectives

________ is a system of goal setting and implementation that involves a cycle of discussion, review, and evaluation of objectives among top and middle managers, supervisors, and employees. Objective planning Management by objectives The three step planning system Cyclical objective analysis

Bureaucratic

______________ organizations are characterized of many layers of management and function with well-defined rules and procedures.

In today's quality control programs, emphasis is placed on:

achieving customer satisfaction.

A marketable security is a cash equivalent unlike _______. a. a treasury bill b. trade credit c. a money market mutual fund d. commercial paper

b. trade credit

A(n) ____ opinion is issued if an external auditor does not find any problems with the way a firm prepares and presents its financial statements. a. quality b. unqualified c. adverse d. corrective

b. unqualified

Chris Callier and a few of his relatives own all of the stock in the Double C Corporation. Double C stock is not sold to the general public. Chris apparently owns stock in a(n)

closed corporation

Return on Sales ratio

divide net income after taxes by net sales

Demographic segmentation.

dividing the market into segments based on variables such as age, life-cycle stage, gender, income, occupation, education, religion, ethnicity, and generation

The preparation of financial statements for people outside the firm (creditors, unions, suppliers, and others) is the goal of:

financial accounting.

One of the hurdles small businesses face when deciding to export is:

financing is often difficult to find.

Establishing a formal code of ethics is:

growing in popularity.

Which of the following is a factor of production?

knowledge

The significant increase in consumer demand following World War II marked the beginning of the:

marketing concept era.

Operations managers at UCON Industries Inc. agree that the company is trying to handle too many supply chain functions on its own, resulting in an inefficient use of resources. This suggests that the company may soon adopt the strategy of: a. acquisition b. outsourcing process c. vertical integration d. cannibalization

outsourcing process

Entrepreneurs often discover that there are many legal and financial issues involved in starting their business. The best way for the small business owner to resolve these issues and protect the firm's resources is:

seek expert guidance and assistance.

Owners equity

total liabilities subtracted from total assets

Equal the equilibrium price

When prices are free to adjust over time, in the long run, the market price of a good tends to: Rise above the equilibrium price in the long run. Equal the equilibrium price. Fall below the equilibrium price in the long run. Have no specific relationship to the equilibrium price.

equal in the equilibrium price

When prices are free to adjust over time, int eh long run, the market price of a good tends to: ______________________________

expect an ownership stake in the company in exchange for financial backing.

When seeking financial backing from a venture capitalist, a small business owner should realize that the venture capitalist will:

learning from others and gaining relevant business experience improves the odds for success.

When starting a small business, it's important to remember: an affinity for taking large amounts of risk is the critical factor in achieving small business success. doing things differently from previous entrepreneurs is the real key to success. learning from others and gaining relevant business experience improves the odds for success. to avoid burnout, limit the number of hours you work each week as you adapt to the rigors of self-employment.

No longer sold to investors on the open market

When investors successfully take a firm private, the firm's stock is: Converted into bonds. Converted into cash. No longer sold to investors on the open market. Pledged as collateral to its bondholders.

franchise agreement

A(n) ___________ is an arrangement whereby someone with a proven idea for a business sells the rights to use the business model, to sell a product or service to others in a given territory.

channel of distribution

A(n) ____________ consists of the marketing intermediaries that transport and store goods as they move through their path from producer to final user. channel of distribution marketing network input-output matrix mode of distribution

entrepreneur

A(n) ____________ is a person who assumes the risk of starting a business. Manager entrepreneur private investor Stakeholder

75 percent

About __________ of the cost of things we buy are marketing costs that go to pay for distribution costs of intermediaries. 25 percent 30 percent 75 percent 90 percent

accepting accountability for the mistakes of the business.

Being your own boss means:

Services differ from production of manufactured good in all the ways except that services

Are not as important as manufactured production to the US economy

Examples of current assets

Cash, Marketable securities, notes receivable and merchandise

Shares of stock cannot be purchased on any stock Exchange or just by any individual. What kind of Corporation is this

Closed Corporation

Taft-Hartley Act.

Closed shops were declared illegal by the: Taft-Hartley Act. Norris-LaGuardia Act. National Labor Relations Board in 1929. U.S. Supreme Court in 1946.

Payment representing some percentage of sale revenue

Commission

Initial Public offering

Common Stock for the first time

__________ involves the production of private-label goods by a foreign company to which a domestic company then attaches its own brand name or trademark.

Contract manufacturing

In the management hierarchy, _____ includes managers who set the overall direction of a firm, articulating a vision, establishing priorities, and allocating time, money, and other resources. a. top management b. middle management c. first-line management d. supervisory management

a. top management

Vieorien, a Swedish company, has allowed Loineker, an American company, the rights to produce Vieorien's products and use the Vieorien trademark. However, Vieorien does not have the authority or the rights to dictate the business operations of Loineker. In this scenario, Loineker is the _____. a. lessor b. lessee c. licensee d. licensor

c. licensee

The primary reason many people become entrepreneurs is to: a. avoid difficult challenges b. avoid risk uncertainty c. make more money d. revolutionize business models

c. make more money

During the _____, businesses began to develop brands to help customers understand the differences among various products. a. industrial revolution b. entrepreneurship era c. marketing era d. relationship era

c. marketing era

Income statement

Earning statement

An organization of Nations formed to promote the free movement of resources and product among its members and to create common economic policies

Economic community

more support of

Empowering employees to better serve customers requires __________ front-line employees. fewer less training for more support for uniforms for

how to determine what should be measured and how to calculate the effect on society.

One problem with conducting a social audit is: that most businesses are really not very concerned with these issues. how to determine what should be measured and how to calculate the effect on society. that the primary focus of management is on the interests of stakeholders. governmental "red-tape."

cut spending

One way to lessen the U.S. government budget deficit is to: cut spending. increase spending. call-in government savings bonds. increase government social programs such as welfare, food-stamps, and Medicare.

Adam Smith believed that since businesspeople are motivated to improve their own prosperity, their actions are unlikely to benefit society as a whole.

False

American businesses that produce high quality goods never experience problems selling goods in countries that set low tariffs.

False

The primary concern of micropreneurs is to balance the risk of starting your own business with the desire to build a powerful organization earning significant profits.

False

a lucrative market

People between the ages of 65 and 74 represent: the unemployed generation the single-parent family generation a lucrative market a group that has never contributed to social security

a job

People need to make a living. Under this circumstance, one of the best social programs a firm can offer is:

technological advances such as the Internet and PayPal for safely fulfilling orders

Increased participation in small business exporting owes credit to _________.

Convenience

Inexpensive, frequently purchased item; buyers exert minimal effort

A general rise in the level of prices is called

Inflation

Publicity

Information about the company its employees and its product that is published or broadcasted in a mass media free of charge

Goal of promotion

Informs Persuades Reminds

investors using private company information to further their own fortunes.

Insider trading involves: an Internet activity that establishes a barter exchange system between businesses. investors using private company information to further their own fortunes. the exchange of assets between companies in the same industry. a payment or reward for socially conscious behavior.

their employer's existing human, financial, and physical resources

Intrapreneurs make use of __________ to develop and launch new products and generate new profits.

Stages of Product Life Cycle

Introduction Growth Maturity Decline

Venture Capital Firms

Invest in small firms that have potential to be very successful

Periodically employees at Southwest Airlines complete satisfaction surveys that management uses to improve employee job satisfaction. What is this example of

Job analysis

The part of a business report that presents suggestions on how to solve a problem

Recommendation

A list of specific qualifications necessary for a particular position is called a

Specification

risk

Starting a business always involves ________. profit loss revenue risk

a general mistrust between workers and management.

When managers disregard ethical concerns, the likely result is:

supply chain.

Taken together, all of the organizations that move goods from the sources of raw materials to ultimate consumers is known as a: multifunction network. supply chain. critical distribution path. marketing priority network.

economic and legal

Taxes and government regulations are part of the ________________ environment of business. economic and legal competitive social technological

American Federation of Labor (AFL)

The __________ was set up as a federation of many separate craft unions. Knights of Labor Committee of Industrial Organizations League of Unions American Federation of Labor (AFL)

Data

The basic resource from which plans are developed and decisions are made

a company report that measures the firm's social contributions inside and outside of the firm.

The social audit is: a company report that always include a net social contribution. a government mandated report required of all firms who have a business license. a company report that measures the firm's social contributions inside and outside of the firm. a recent development that creates a ratio to compare a firm's corporate philanthropy to its profits.

demand-oriented pricing.

The strategy of charging prices based on consumer demand rather than cost or a predetermined profit margin represents: competition-based pricing. cost-based pricing. target orientation. demand-oriented pricing.

target costing.

The strategy of first determining what the market is willing to pay, then subtracting a desired profit margin to determine a desired cost of production is called: cost-based pricing. target costing. penetration pricing. skimming pricing.

Disadvantages of

Unlimited Liability Disagreements Difficulty in withdrawing from agreement Lack of Continuity

hire strikebreakers

When workers go on strike, management is often compelled to keep the business operation running. If it is unable to achieve this on its own, management may: elect to proceed with a lockout. close down the facility and permanently fire the strikers. hire strikebreakers. picket.

expect socially responsible behavior from their business partners

When working with international firms, U.S. businesses:

company leaders can threaten competitors and minimize competition.

When corruption is present in the business environment: company leaders can threaten competitors and minimize competition. factories are easier to build without a government permit. it promotes fair competition. laborers can make more money.

Effective managers often use a variety of leadership styles.

Which of the following statements about leadership styles is most accurate?

Rack jobbers

___________ are limited-function wholesalers who furnish shelves full of merchandise to retailers who sell these items on consignment. Cash-and-carry wholesalers Rack jobbers Merchandise forwarders Consignment agents

Microeconomics

____________ looks at the behavior of individual people and organizations in specific markets. Macroeconomics Finite economics Microeconomics Nanoeconomics

A monopoly

_________________________ exists when the entire supply of a good is controlled by a single seller

When the market price of a good is above its equilibrium value:

a surplus will exist in the market.

Hierarchy

a(n) __________ consists of one person at the top of the organization and many levels of managers who are responsible to that person. Hierarchy Oligarchy inverted organization informal organization

Which of the following is the similarity between venture capital firms and angel investors? a. Both invest in start-up companies with high growth potential in exchange for a share of ownership. b. Both charge very high interest rates and structure loans to be repaid in months rather than years. c. Both require an entrepreneur to provide a minimum of 25 to 30 percent of the start-up costs. d. Both only finance large corporations that operate in universal and generic markets.

a. Both invest in start-up companies with high growth potential in exchange for a share of ownership.

Which of the following is uncharacteristic of a capital budgeting proposal evaluation process? a. Buying Treasury bills and bonds for the next six months b. Replacing computer equipment with upgraded models c. Buying new equipment to increase the capacity of the production line d. Building a new factory, warehouse, or office building

a. Buying Treasury bills and bonds for the next six months

______ is the ongoing process of acquiring, maintaining, and growing profitable customer associations by delivering unmatched value. a. Customer relationship management b. Customer satisfaction c. Customer segmentation d. Customer observation

a. Customer relationship management

In a(n) ________, all of the members are skilled specialists in a particular trade.

craft union

Mini-Case Peppy Roaney believes that pizza is a much more versatile dish than most people realize. As the national marketing manager for Pizza Guys, he observed how consumers were delighted with barbecue pizza, Hawaiian pizza, Mexican pizza and other unique combinations. His suggestions for other unusual pizzas, however, were met with a negative response by his superiors. Even some of his colleagues ridiculed his ideas. Peppy's frustration has continued and he is now in the planning stages of starting his own business, Pizza Whirl. Peppy realizes that he will be taking a risk when he leaves his job with Pizza Guys and starts out on his own. However, Peppy is excited about the potential of profitably catering to the unmet desires of pizza consumers. Peppy is confident that Pizza Whirl will be a success, but only if he has enough money to do things right. Since Peppy has very little personal wealth, he knows he must obtain financial help. In order to be prepared for meetings with bankers and potential investors, Peppy should:

develop a business plan.

When prices are free to adjust over time, the market price of a good tends to:

equal the equilibrium price in the long run.

The "Spotlight on Small Business" box in Chapter 14 suggests the product offered by Dream Dinners is the:

experience of having a good time preparing meals with others.

An entrepreneurial team consists of:

experienced managers from different areas of business

The threat of terrorism __________ the cost of operating a business.

increases

FIFO and LIFO are two common:

inventory valuation methods.

Insider trading involves

investors using private company information to further their own fortunes

Brand mark

is a design element, such as a symbol (e.g., Nike swoosh ), logo (e.g., Yahoo! graphic), a character (e.g., Keebler elves) or even a sound (e.g., Intel inside sound), that provides visual or auditory recognition for the product.

The financial manager of Carolina Graphics negotiated a ________ with her bank that allows Carolina to borrow up to $50,000 without collateral. This arrangement eliminates the need to renegotiate the terms of the loan and complete new paper work each time Carolina borrows money. The preapproved loan agreement is contingent upon the bank having the funds available.

line of credit

The management of charities, government agencies, and schools is ________ the management of a business.

much the same as

Business incubators are government-funded facilities intended to:

offer startup firms low-cost office space and basic business services.

Global franchising offers:

opportunities for both large and small franchises.

The danger in writing new laws to correct behavior is that:

people may begin to think that any behavior that is within the law is acceptable.

After retiring as an oil industry executive, Paul was asked to join SCORE. As a SCORE counselor Paul will:

provide consulting services to small businesses for free.

A major purpose of all of accounting is to:

provide financial information that is useful to decision makers.

In multilevel marketing, salespeople have an incentive to:

recruit new salespeople, because they receive a commission on the sales made by the people they recruit.

Product screening is designed to:

reduce the number of new-product ideas being worked on at any one time.

Many small businesses rely on factoring as a source of short-term financing because:

small firms often find it difficult to qualify for bank loans.

Trade Credit

Least expensive form of short term financing

Franchising Advantages

Less Risk Training and Support Brand Recognition Access to Funding

This person invest in the business but has no management responsibility

Limited partner

A detailed written statement that describes the nature of the business, the target market, the firm's competitive advantages, as well as the owner's resources and qualifications is a:

business plan.

When a person buys stock through an online broker such as Charles Schwab, he or she participates in ______ e-commerce. a. business-to-business b. business-to-government c. business-to-consumer d. consumer-to-consumer

business-to-consumer

A review of small business successes and failures indicates that the:

businesses with the potential of high profit levels tend to be difficult to start and hard to keep going.

Which of the following would be helpful in explaining why individuals accept the risks of business ownership?

the sense of self-satisfaction and independence

Any debts or damages incurred by a firm organized as a sole proprietorship are:

the sole responsibility of the owner.

concern for the welfare of society.

1. Corporate social responsibility describes the firm's: exclusive responsibility to its stockholders. ability to plan for the unexpected. commitment to a management training program. concern for the welfare of society.

is the appearance of being "green", when in fact the business is only making green claims without genuinely demonstrating a commitment to going green.

1. Green washing: is the most popular sub-movement of the green movement. clears the path for a firm to develop products and services that benefit society. is the appearance of being "green", when in fact the business is only making green claims without genuinely demonstrating a commitment to going green. changing one's entire product line to only offering green products.

Regulation FD (Fair Disclosure)

1. In order to avoid insider trading, the SEC (Securities and Exchange Commission) has adopted a new rule called _____________ that requires publicly traded companies to release information to everyone in a timely manner, rather than just to a few persons. Insider Trading Phenomenon Regulation FD (Fair Disclosure) Red Herring Regulation PDI (Public Display of Information Act)

corporate social initiatives

1. In recent years, progressive firms have embarked upon ___________, where they will commit company resources and expertise toward helping-out in emergency type situations. corporate philanthropy corporate social activism social civility corporate social initiatives

should be based on fairness, honesty, openness, and moral integrity.

1. Relationships between businesses and among nations, should be based on international law, exclusively. should be based on fairness, honesty, openness, and moral integrity. should be scrutinized carefully to make some determination as to whether both sides have the same type of leader behavior when it comes to assigning work. will be adversarial because seldom does either side believe in win-win situations.

Limited liability companies, which do not have the restrictive eligibility requirements of S corporations and offer greater flexibility in the choice of tax treatment, are now legal in all 50 states.

1. The S corporation is likely to be less popular in the future because: Congress repealed the limited liability protection of S corporations and limited them to companies with earnings of less than $3 million per year. Limited liability companies, which do not have the restrictive eligibility requirements of S corporations and offer greater flexibility in the choice of tax treatment, are now legal in all 50 states. Many states significantly increased the annual fee that S corporations must pay to maintain their tax status, thus eliminating the financial advantages of this form of ownership. S corporations have been made illegal in several states as a reaction to widespread abuse of the special benefits available to this type of business.

it is a theoretical model that may not be followed by all products. it is a theoretical model that may not be followed by all products.

1. When reviewing the product life cycle model, it is important to remember that: although all products go through each stage of the life cycle, the time frame can vary considerably from one product to another. it is a theoretical model that may not be followed by all products. the maturity stage accounts for the fastest growth in sales. companies earn their largest profits in the introduction stage.

Rico just bought an expensive hi-def television and is already worried that he could have found a better deal if he had shopped more carefully.

1. Which of the following is the best example of cognitive dissonance? John has been shopping for a new television set, but has not yet made a purchase because he is having trouble deciding which brand to buy. Julie decided to eat at a restaurant she previously avoided after learning that several movie stars she admires endorse the restaurant. Marquetto is torn between going to the movies with friends or staying home and studying for an exam tomorrow. Rico just bought an expensive hi-def television and is already worried that he could have found a better deal if he had shopped more carefully.

Does business in one or more states, but is chartered in another state.

1. Which of the following statements is the most accurate? A foreign corporation: Does business in one or more states, but is chartered in another state. Is 50% owned by individuals or companies from another nation. Is headquartered in another nation. Is the same thing as a multinational corporation.

Distribution Channel

A sequence of organizations that directs a product from one producer to the ultimate user

expectancy theory

According to __________ the amount of effort employees devote to a task depends on their expectations of the outcome. scientific management theory the feedback principle the linkage effect expectancy theory

Which of the following is not the four states of the business cycle

Deficit The Business Cycle 1. Trough 2. Expansion 3. Peak 4. Recession

create a detailed business plan to help the bankers and investors understand why this business will succeed.

Entrepreneurs seeking financial support from bankers and potential investors should:

The utility created by converting production inputs into finished product is called what utility

Form utility

Which of the following questions can help us avoid a decision that would injure our self-esteem?

How will it make me feel about myself?

Can be reconciled by the fact that manufacturing is very efficient and requires fewer workers to produce the same amount, or even more output.

Manufacturing output in the U.S. continues to increase. The high unemployment of skilled workers who previously worked in the manufacturing sector ________________________. Is the result of untrained laborers. Is strictly a factor of U.S. manufacturing companies outsourcing the majority of their work. Can be reconciled by the fact that manufacturing is very efficient and requires fewer workers to produce the same amount, or even more output. Can be attributed to manufacturing going "green."

Cash Flow

Movement of money into and out of

A firm that operates on a worldwide scale is called

Multinational Enterprise

As a prospective entrepreneur, Gregory benefited from his experience working for an established florist. He now feels that he is ready to start his own shop, but he lacks the necessary capital. Which of the following would best help Gregory achieve his goal of business ownership?

Offer to take over management of the business, allowing the owner to enjoy some leisure time.

A company is a Electronics manufacturer that purchase wires from various wire manufacturer. This company is a part of which type of Market

Producer

form utility

Production creates: time utility. possession utility. form utility. marginal utility.

Many economists refer to the average level of output per worker as

Productivity

A technique used to gather enough stockholder votes to control a Target company is known as a

Proxy fight

Audit

Required by many bankers and lenders who are trying to validate a company accountant state

determine what rewards are valued by employees.

Researchers David Nadler and Edward Lawler suggest that in order to use expectancy theory to improve worker motivation, the first thing managers should do is: identify the critical path for the project. determine what rewards are valued by employees. ensure that the rewards are tied to the employee's performance. decide on a way to monitor employee performance.

Protective tariff

Responding to pressure from political lobbyists representing the U.S. steel industry, the U.S. government levies a tax on steel products imported from Europe. This is an example of a(n): protective tariff. revenue tariff. quota. embargo.

specialty stores.

Retailers such as jewelry stores, shoe stores, and bicycle shops are: department stores. discount stores. warehouse clubs. specialty stores.

a large trade deficit with China.

Reviewing the recent experience of the United States in global markets reveals that the U.S has: a merchandise trade surplus with the rest of the world. a large trade deficit with China. avoided becoming a debtor nation. attracted very little foreign direct investment.

operational

Strategic planning looks at the organization as a whole, while ________ planning focuses on daily tasks and decisions.

Examples of Hinesburg hygiene factors

Supervision pay working condition NOT Responsibility

Elton Mayo

The Hawthorne studies were conducted by ____________ and his colleagues from Harvard University. Henry Gantt Elton Mayo Peter Drucker John Hawthorne

trademark

The Pillsbury Doughboy is an example of a: patent. trademark. private brand. generic label.

An internal Force that's not affecting businesses

The egg

a failure of leadership to establish ethical standards.

The majority of CEOs blame unethical employee conduct on: the breakdown of traditional religious institutions. a failure of leadership to establish ethical standards. the increase in lawsuits used to avoid personal responsibility. the nation's business schools that tolerate unethical behavior in students.

mixed economy

There are some restrictions on freedom of assembly and freedom of speech under a ____________ system. capitalist Socialist mixed economy communist

After losing her manufacturing job in the U.S. automobile industry, Juanita has found that well paid service sector jobs require retraining and further education.

True

small businesses can serve niche markets that larger firms do not serve well.

Using the analogy of boulders vs. sand for filling a hole, the textbook points out that: small businesses are employing a record number of employees because they have to work twice as hard at producing the same amount of output as a large competitor. small businesses can serve niche markets that larger firms do not serve well. the sand represents the small businesses that start and fail each year, while the boulders represent the large companies whose resources permit them to stay the course. large companies serve market niches so much better than small companies.

They look at problems and see opportunities.

Which of the following statements is characteristic of entrepreneurs?

Line organizations

Which of the organization models follows all of Fayol's traditional management rules? Line organizations Line-and-staff organizations Cross-functional teams Matrix

Technology affects people in

all industries

While the United State exports the largest __________ of goods globally, it exports a much lower __________ of its products than many other countries do.

amount; percentage

Francine is a bright and talented woman. She recently quit her job at a major corporation because she and her boss didn't get along and she saw little chance for advancement. Francine has not yet found another job, but the economy appears healthy and lots of firms are hiring people with her qualifications. Thus, Francine is confident that she will find a job in the near future. Which of the following statements about Francine's current situation is most accurate? Francine is:

an example of frictional unemployment

The Federal Reserve System operates

as a semi-private organization not under the direct control of the government.

Resources that a firm owns are called:

assets.

Today, packaging is:

assuming an increasingly important role in the promotion of products.

In operations management, quality is defined in terms of: a. the ration of cost to output b. how well a product satisfies customer preferences c. the number of defects per thousand unit of a product d. the time taken to inspect one unit of the product

b. how well a product satisfies customer preferences

The capital expenditure budget: a. provides quarterly estimates of the number of units of each product a firm expects to sell b. identifies a firm's planned investments in major fixed assets and long-term projects. c. identifies short-term fluctuations in cash flows that display cash deficits and surpluses. d. contains the budgets for direct labor, direct materials, and overhead costs

b. identifies a firm's planned investments in major fixed assets and long-term projects.

Microeconomics focuses on: a. the employment rate in a country. b. individual businesses. c. the gross domestic product of a country. d. taxation policies.

b. individual businesses.

Which of the following statements is true of the Financial Accounting Standards Board (FASB)? a. Its authority comes for the Internal Revenue Service b. It consists of five members appointed by the Federal Trade Commission (FTC) c. Its members can be reappointed to serve one additional term d. Its members can retain ties with firms in which they were previously employed

c. Its members can be reappointed to serve one additional term

Which of the following is a major challenge faced by most human resource departments? a. Restrictions from top management in making strategic contributions. b. Inability to find the root cause of problems in most situations. c. Being considered irrelevant by many employees. d. Receiving less pay.

c. Being considered irrelevant by many employees.

Jessica, the chief human resources officer at GA Trading Company, is facing a problem at work. One third of the employees of the company, who belong to the Baby Boomer generation, will be retiring next year. In response to this problem, she introduces benefits like flextime and telecommuting. In this scenario, she is trying to retain _____. a. younger workers b. female workers c. older workers d. foreign workers

c. older workers

Franchise systems are one type of __________ distribution system.

contractual

Cole González is the owner of Timmy's Treats, a small company that makes organic dog treats. Each year Cole donates a portion of his profits to the Dumb Friends League so they can care for animals without homes. Cole is practicing:

corporate social responsibility

One disadvantage of _________ is the initial cost of starting this form of business organization.

corporations

Firms that have adopted _________ seek to do more than simply sell a product; they seek to learn as much as possible about their customers in order to satisfy them with goods and services over time.

customer relationship management

The idea behind the Deming Cycle is to:

find potential errors before they occur.

In global trading, the United States ranks _______ in terms of the volume of exports.

first

Which of the following statements about the impact of CAD/CAM on manufacturing is most accurate? CAD/CAM:

has made it possible to produce custom-designed products with little increase in costs.

The U.S. government has announced a 5 million-pound annual limit on beef imported from Argentina. This type of trade restriction is called a(n):

import quota

Asset

property owned by a person or company, regarded as having value and available to meet debts, commitments, or legacies.

In a sole proprietorship, the profits earned by the business are:

the property of the owner, except for taxes owed to the government.

According to information contained in the "Reaching Beyond Our Borders" box in Chapter 12, a major reason the nations belonging to the European Union are experiencing wage tension is that:

the acceptance of the euro as the common currency for the nations of the European Union has made the large differences in wage rates in the various countries more obvious.

Sweet Dream Confectioners uses the same ingredients as most other producers of chocolate candies. In fact, taste tests suggest that the candy itself, while quite good, isn't much better than other well-known brands of chocolate. However, the company wraps its candies in expensive looking foil and places them in very attractive boxes. It promotes its products in advertisements as "the ultimate in chocolate experience." Sweet Dream charges a much higher price than most competitors, but sales continue to grow. This success indicates that:

there is more to the total product offer than the physical product.

Jacob is in urgent need of financing for his rapidly-growing online travel business, Travelmuch. Since Jacob does not usually pay off his loans on time, his credit score is very low. He feels overwhelmed and thinks that it would be ideal if he gets some guidance on how to run the business. Given his situation, which of the following key sources of funding would be ideal for Jacob? a. A commercial bank loan b. A venture capital firm c. An initial public offering d. A Small Business Administration (SBA) loan

b. A venture capital firm

Which of the following statements is true of expectancy theory? a. It suggests workers exercise self-direction and self-control to meet company goals. b. Employees will not be motivated if their efforts do not lead to reward. c. It suggests that an employee can be punished based on poor performance. d. Workers will not be motivated if their reward is lesser than other workers' rewards.

b. Employees will not be motivated if their efforts do not lead to reward.

Which of the following is a drawback of buying a franchise? a. Half of the franchisees of a brand fail in their first year of operations. b. If the national brand suffers, so does the franchisee. c. Getting to know the business model may take time. d. the buyer has to finance the operation of the franchise.

b. If the national brand suffers, so does the franchisee.

Which of the following statements is true of computer-integrated manufacturing (CIM) software? a. It is the result when computer-aided engineering software is integrated with a computer-aided manufacturing application. b. It allows firms to produce custom-designed products for consumers quickly and at a low cost. c. It prevents the design and production process of products from being highly automated. d. It does not allow firms to implement the mass customization strategy.

b. It allows firms to produce custom-designed products for consumers quickly and at a low cost.

Which of the following was a major outcome of the Banking Act of 1933? a. It established the Federal Reserve as the central bank in the U.S b. It banned commercial banks from competing with non-depository institutions c. It prohibited fraud in the sale of newly issued stocks and bonds d. It created the Securities and Exchange Commission to oversee the securities industry

b. It banned commercial banks from competing with non-depository institutions

Identify a true statement about global trade. a. It increases a firm's dependence on its domestic economy. b. It offers companies an invaluable source of new ideas. c. It increases the economic risk of multinational companies. d. It reduces a firm's opportunity to tap into growing new markets.

b. It offers companies an invaluable source of new ideas.

In the context of human resource management challenges, which of the following statements is true of older workers? a. Older workers have no expectation that their employers will be loyal to them, and they don't feel that they owe their companies strong loyalty. b. Many companies have started flexible schedules, training opportunities, and creative pay schedules to retain their best older employees c. Older workers have startlingly high expectations for their pay, their responsibilities, and job flexibility, but little willingness to "pay dues." d. Unlike younger workers, older workers have a proven ability to multitask and a multicultural perspective.

b. Many companies have started flexible schedules, training opportunities, and creative pay schedules to retain their best older employees.

_____ refers to Federal Reserve decisions that shape the economy by influencing interest rates and the supply of money. a. Fiscal policy b. Monetary policy c. Revenue policy d. Deficit policy

b. Monetary policy

When a large company downsizes, some of the middle-aged and older workers who are laid off find it difficult to get another job. When such workers decide to start their own businesses, they are known as _______. a. Welfare recipients. b. Necessity entrepreneurs. c. Post-retirement workers. d. Baby boomers.

b. Necessity entrepreneurs

Established by the Sarbanes-Oxley Act of 2002, it is the responsibility of the _____ to protect the interests of investors and further the public interest in the preparation of informative, fair, and independent audit reports. a. Private Company Accounting Oversight Board b. Public Company Accounting Oversight Board c. Public Enterprise Accounting Regulation Agency d. Private Enterprise Accounting Regulation Agency

b. Public Company Accounting Oversight Board

_____ refer to the pay that employees receive over a fixed period, most often weekly or monthly. a. Commissions b. Salaries c. Wages d. Bonuses

b. Salaries

_______ include differences among countries in language, attitudes, and value. a. Economic differences b. Sociocultural differences c. Political differences d. Legal differences

b. Sociocultural differences

_____ refers to unsolicited commercial emails, usually sent to a large number of people with little regard to the users' interest in the product or its promotion. a. Cyberware b. Spam c. Opt-in mail d. Malware

b. Spam

Iris has been told by her boss that the company is planning to lay off some people and that only one employee in her department will be retained. In order to keep her job, Iris has to sabotage the professional image and good work of her colleagues or she will be laid off. This situation is an example of _____. a. philanthropy b. an ethical dilemma c. moral absolutism d. a company audit

b. an ethical dilemma

Documents stored on the World Wide Web are written and linked together using ____. a. decision support systems (DSS) b. computer-aided design (CAD) c. expert systems (ES) d. Hypertext Markup Langauge (HTML)

b. computer-aided design (CAD)

The _____ expresses the value of one nation's currency in terms of another nation's currency. a. liquidity ratio b. exchange rate c. countertrade ratio d. discount rate

b. exchange rate

The core goal of successful firms is to: a. create a diverse workforce in an organization. b. generate long-term profits by delivering unsurpassed value to their customers. c. meet the rising expectation of workers. d. ensure free trade of goods and services within local and international markets.

b. generate long-term profits by delivering unsurpassed value to their customers.

Which of the following is an intimidating difficulty that small businesses face? a. obstacles in technology b. high health insurance costs c. high overhead costs d. lack of customer service

b. high health insurance costs

Critics of the North American Free Trade Agreement (NAFTA) believe Mexico and Canada have contributed to the United States' trade deficit because: a. the economies of these nations have grown faster than the economy of the U.S. b. imports from Mexico and Canada have grown faster than exports to these nations. c. import and export transactions with Mexico and Canada have remained static. d. the U.S. imports goods from China and India rather than Mexico and Canada

b. imports from Mexico and Canada have grown faster than exports to these nations.

Which of the following business product categories refers to large capital purchases designed for a long productive life and the marketing of which emphasizes personal selling and customization? a. specialty products b. installations c. unsought products d. accessories

b. installations

According to a report in Fast Company, a major reason why many human resource managers struggle to add value to their firm is that they: a. focus on revenue-earning products rather than on people b. lack business acumen in addition to strategic skills c. are in staff positions rather than line positions d. do not have well-honed communications skills

b. lack business acumen in addition to strategic skills

Requiring red-tape-intensive import licenses for certain categories is an example of a(n): a. embargo b. nontariff barrier c. quota d. voluntary export restraint

b. nontariff barrier

In order to preserve independence and impartiality, Financial Accounting Standards Board (FASB) members are required to: a. serve the board for the rest of their lives once they are appointed b. sever all ties with any firms or institutions they served prior to joining the board c. sign a non-disclosure agreement and hand over all information of their previous companies to the board d. pass a rigorous two-day, four-part examination on major accounting concepts

b. sever all ties with any firms or institutions they served prior to joining the board

______ is often referred to as the world's largest computer network. a. a cloud-based network b. the internet c. an extranet d. the kernel

b. the internet

_____ software enables users to test, analyze, and optimize their designs through computer simulations. a. Critical path manufacturing (CPM) b. Computer-aided engineering (CAE) c. System-aided manufacturing (SAM) d. Computer-integrated manufacturing (CIM)

c. Computer-aided engineering (CAE)

Jessica Marshall, the CEO of ANS Oil Drilling Inc., has decided to set aside $10,000 from the company's annual profit to donate to local nonprofit organizations. This donation is an example of: a. Corporate opportunism b. Stakeholder remuneration c. Corporate philanthropy d. Stakeholder dividends

c. Corporate philanthropy

Which of the following is a duty of a socially responsible organization toward customers? a. Increasing overhead business costs significantly. b. Using strategies such as planned obsolescence for the organization;s growth. c. Delivering value by providing quality products at fair prices. d. Being involved in competitive practices such as collusion and monopoly.

c. Delivering value by providing quality products at fair prices.

David works in a small manufacturing firm that makes customized and ergonomically sound office furniture. He feels it is important to work directly with customers to design high-quality furniture that precisely meets their needs. From the given information, we can conclude that David tends to focus on _____. a. Mass production b. Efficiency c. Effectiveness d. Undifferentiated marketing

c. Effectiveness

_____ means establishing only one retail outlet in a given area. a. Intensive distribution b. Wholesale distribution c. Exclusive distribution d. Direct distribution

c. Exclusive distribution

Which of the following occurred during the entrepreneurship era? a. The balance of power shifted away from producers and toward consumers. b. Web and digital resources were used to gather detailed information about customers c. Industrial titans created enormous wealth and dominated their markets. d. The standard of living declined at a high rate.

c. Industrial titans created enormous wealth and dominated their markets.

Identify a true statement about exporting, a. It is colloquially known as foreign outsourcing. b. It is ineffective for small and mid-sized companies. c. It is the most basic level of international market development. d. It means producing products abroad and selling them domestically.

c. It is the most basic level of international market development.

Identify a true statement about macroeconomics. a. It focuses primarily on monitoring the activities of individual businesses. b. It hardly affects the day-to-day lives of people. c. It is the study of a country's overall economic dynamics. d. It has little or no influence on variables such as jobs and taxes.

c. It is the study of a country's overall economic dynamics.

____ ratios measure the ability of an organization to convert assets into the cash it needs to pay off liabilities that come due in the next year. a. Activity b. Leverage c. Liquidity d. Profitability

c. Liquidity

Insurance company a. They sell securities for higher prices than they paid to purchase them b. They collect premiums from policy holders. c. They act as agents for investors who want to buy or sell financial securities. d. They issue new securities to raise financial capital.

c. They act as agents for investors who want to buy or sell financial securities.

Which of the following is the purpose of a written code of ethics? a. To merely satisfy the requirements of the Sarbanes-Oxley Act. b. To eliminate the need for top management to monitor ethical behavior, thus giving these managers more time to deal with other issues. c. To provide guidance to help employees make sound ethical decisions across a range of situations. d. To explain the rules laid out by regulatory bodies that monitor business firms.

c. To provide guidance to help employees make sound ethical decisions across a range of situation.

Overhead costs for small businesses: a. tend to be much higher than they are for large businesses as a percentage of total expenses. b. are about the same as they are for big companies when measured as a percentage of total expenses. c. are often lower than they are for big companies, giving small businesses a competitive opportunity. d. primarily arise because small businesses must hire many consultants and advisers from outside.

c. are often lower than they are for big companies, giving small businesses a competitive opportunity.

In the context of the U.S. economy, small businesses: a. do not create a significant percentage of new jobs b. contribute to a very small percentage of the gross domestic product c. are often the first to contract when times are tough d. have an insignificant impact of the state of the economy

c. are often the first to contract when times are tough

The overall value of a brand to an organization - the extra money that consumers will spend to buy that brand - is called _____. a. brand extension b. brand mix c. brand equity d. brand segment

c. brand equity

When tax revenue is higher than government expenditures, the government incurs a ____. a. private debt b. trade gap c. budget surplus d. fiscal deficit

c. budget surplus

The two key phases of a business cycle are: a. stagflation and hyperinflation. b. depression and contraction. c. contraction and expansion. d. recession and deflation.

c. contraction and expansion.

Legislation that supports enforceable contracts usually forms part of an organization's ____ environment. a. competitive b. cultural c. economic d. social

c. economic

Financial capital refers to the: a. money that a business earns in sales, minus the expenses. b. costs a business incurs when its expenses are greater than its revenues. c. funds a firm uses to acquire its assets and finance its operations. d. returns that a firm pays to its owners for their investment in the company.

c. funds a firm uses to acquire its assets and finance its operations.

The senior management of a firm recognizes the potential of the human resource (HR) department: a. if the HR department is listed as a line department instead of a staff department on the organization chart b. if the HR department is consulted only when legal problems arise c. if the HR department reports to the CEO of the firm d. if the HR department is spatially close to the senior management

c. if the HR department reports to the CEO of the firm

A(n) ____ is the first time a company issues stock that may be bought by the general public. a. alternative public offering b. reverse public offering c. initial public offering d. direct public offering

c. initial public offering

Annual reports include _____ that disclose additional information about a firm's operations and special circumstances that clarify and supplement the numbers reported on the financial statements. a. subpoenas b. testimonials c. notes d. future projections

c. notes

Which of the following is an example of an internal recruitment method? a. trade associations b. University employment centers c. promotions d. newspaper ads

c. promotions

The main disadvantage of financial leverage is that it: a. increases the taxes of firms that use it. b. requires owners to invest even more of their own money. c. reduces the financial return to stockholders when times are bad. d. requires firms that use it to make higher dividend payments.

c. reduces the financial return to stockholders when times are bad.

At Blyrie Technologies, a leading spare parts manufacturing firm, a few workers were injured on the job. Following this, its management decided to implement certain changes in the workplace. It provided the workers with enough protective gears, health insurance plans, and life insurance. According to Maslow's hierarchy of needs theory, Blyrie Technologies is motivating its employees by meeting their _____. a. self-actualization needs b. physiological needs c. safety needs d. social needs

c. safety needs

Paul, an employee at a well-established bank, offers free financial advice to account holders. Paul is offering a _______. a. good b. product c. service d. process

c. service

In order to access stock information for a particular corporation on a financial website, an individual normally types in the corporation's ____. a. stock earnings per share b. stock number c. stock symbol d. SEC registration number

c. stock symbol

Consumerism is a widely accepted social movement. Proponents of this movement believe that _____ is a right of all consumers. a. the right to the lowest price b. the right to purchase an extended warranty c. the right to be informed d. the right to return a product in seven days for a full refund

c. the right to be informed

Which of the following is a powerful contribution made by small businesses to the U.S. economy? a. vitalizing rural towns b. powering the stock market c. vitalizing inner cities d. accounting for all private sector output

c. vitalizing inner cities

Buccaneer Industries manufactures and markets expensive industrial cranes. This long lasting heavy equipment would be classified as a(n) __________ good.

capital

Notes of appreciation, awards, and gift certificates given to reward employees for accomplishing or surpassing established objectives are all examples of:

cashless bonuses.

The _____ of 2010 expanded the Fed's regulatory authority over nondepository financial institutions, such as hedge funds and mortgage brokers that had previously operated with little regulatory oversight or accountability a. Sarbanes-Oxley Act b. Dodd-Frank Act c. Glass-Steagall Act d. Gramm-Bliley-Leach Act

d. Dodd-Frank Act

______ is a branch of accounting that addresses the needs of external stakeholders, including stockholders, creditors, and government regulators. a. Forensic accounting b. Investigative accounting c. Managerial accounting d. Financial accounting

d. Financial accounting

Which of the following is a difference between goods and services? a. Goods must be consumed when they are produced, while services can be stored in an inventory. b. Goods can be "experienced" and don't have a physical form, while services have a physical form and can be seen, touched, and handled. c. Goods must be consumed where they are provided, while services can be shipped. d. Goods are produced independently of the consumer, while services often require customers to be actively involved in their production.

d. Goods are produced independently of the consumer, while services often require customers to be actively involved in their production.

Which of the following statements is true of business-to-business (B2B) e-commerce? a. It involves much less negotiation than business-to-consumer (B2C) e-commerce b. It involves businesses selling supplies, components, machinery, equipment, or services to individual final consumers c. It involves much less investment than business-to-consumer (B2C) e-commerce d. It involves business selling supplies, components, machinery, equipment, or services to other businesses

d. It involves business selling supplies, components, machinery, equipment, or service to other businesses

Which of the following statements is true of the distribution strategy? a. It enables physical distribution without the interference of channel intermediaries. b. It aims at eliminating channel intermediaries. c. It reconciles conflicts between pricing strategies and promotion strategies. d. It is a plan for delivering the right product to the right person at the right place at the right time.

d. It is a plan for delivering the right product to the right person at the right place at the right time.

____ is a technology that stores information on a microchip - such as a unique serial number - and transmits it to a reader when it is within range. a. Global positioning system b. Computer-aided design c. Graphical user interface d. Radio frequency identification

d. Radio frequency identification

Who among the following is most likely to benefit in a case where there is a strong dollar against a euro? a. Hailinser, an American who exports goods to Europe b. Fieolia Corp., a European firm that imports goods from America c. Joinieker Inc., a Europea firm with American operations d. Ron, an American who is touring Europe

d. Ron, an American who is touring Europe

Every year, the members of the managing committee at Willem Inc. systematically evaluate how well the firm meets its ethics and corporate responsibility objectives. This process is referred to as a(n) _____. a. Sustainable development b. Planned obsolescence c. Job deskilling d. Social audit

d. Social audit

_____ is software that installs itself on a computer without permission and then tracks its behavior in some way. a. Adware b. Screenware c. Freeware d. Spyware

d. Spyware

Which of the following statements is a characteristic of entrepreneurs? a. They are hardly interested in created wealth for themselves. b. They create value for themselves at the expense of others. c. They have little or no impact on other people in the society. d. They product a ripple effect that enriches everyone around them.

d. The produce a ripple effect that enriches everyone around them.

Which of the following statements is true of angel investors? a. They usually expect a minimum return on investment from the start-ups they fund. b. They typically incest in low-risk opportunities involving established firms. c. They usually fund more mature firms that have an established tract record. d. They typically provide funds to start-ups in exchange for a share of ownership.

d. They typically provide funds to start-ups in exchange for a share of ownership.

Which of the following is the goal of sustainable development a. To increase sales of a company's products and raise money in order to donate a significant amount to nonprofit organizations. b. To design products to fail in order to shorten the time between purchases, thus spiking the repurchase profits. c. To report unethical and illegal activities of employees to the relevant authorities or the media. d. To meet the needs of the current generation without harming the ability of future generations to meet their needs.

d. To meet the needs of the current generation without harming the ability of future generations to meet their needs.

____ is best described as an Internet advertising strategy that attempts to get customers to communicate a firm's message to friends, family, and colleagues. a. Phishing b. Supply-chaining c. E-tailing d. Viral marketing

d. Viral marketing

Which of the following can be achieved by marketers by acquiring customer loyalty? a. increased actual value of products b. ability to function without feedback c. automatic market segmentation d. acquisition of new customers

d. acquisition of new customers

The ______ forecasts the types and amounts of assets a firm will need to implement its future plans as well as the amount of additional financing the firm must arrange in order to acquire those assets. a. query report b. cash budget c. statement of cash flows d. budgeted balance sheet

d. budgeted balance sheet

A _______ is a formal document that describes a business concept and outlines the core business objectives while detailing the strategies and timelines for achieving those objectives. a. business contract b. business commission c. business directory d. business plan

d. business plan

While raising start-up funds from friends and family, it is: a. acceptable to repay them b. essential to involve them in company dealings c. acceptable to not document the financing d. essential to keep the relationship as professional as possible

d. essential to keep the relationship as professional as possible

Firms create ____ by offering limited access to certain areas of their intranets to select groups of people. a. open systems b. expert systems c. horizontally integrated networks d. extranets

d. extranets

People with an external locus of control: a. are extremely self-reliant b. are more likely to take responsibility for their own failure c. feel that they have complete control over their lives d. feel buffeted by forces such as random luck

d. feel buffeted by forces such as random luck

Quitor Inc., which is based in the U.S., contracts with Falconnion Suppliers, a small-scale supplier in Taiwan, to manufacture its computers and tablets across the world. This strategy by Quitor Inc. is an example of ______. a. joint venture b. franchizing c. exporting d. foreign outsourcing

d. foreign outsourcing

A(n) _____ is a financial intermediary that specializes in helping firms raise financial capital by issuing securities in primary markets. a. credit union b. commercial bank c. savings and loan association d. investment bank

d. investment bank

The benefit that cloud computing provides to individual users is that: a. it allows them to access their data from any computer without an Internet connection b. it requires only one major investment for the security of their data. c. it allows them to communicate with their friends and relatives in any part of the world without being connected to the Internet d. it allows them to store large files without taking up valuable space on their computers' hard drives.

d. it allows them to store large files without taking up valuable space on their computers' hard drives

Sebastian is an employee of Power Trans Inc. His duties include preparing reports and analyzing company accounting data. He also appraises financial performances, verifies the accuracy and validity of the company's internal records and accounting procedures. Sebastian's role is that of a _____. a. forensic accountant b. government auditor c. certified fraud examiner d. management accountant

d. management accountant

The two broad categories of software are: a. text software and data software. b. entertainment software and business software. c. resident software and extrinsic software d. system software and application software

d. system software and application software

Computers, digital tools, and telecommunications are most likely to be part of the ____ dimension of a business environment. a. economic b. social c. cultural d. technological

d. technological

Entrepreneurs find new solutions to old problems and develop new products that people didn't even realize they needed. These entrepreneurs are said to have _____. a. hindsight b. uncertainty avoidance c. an external locus of control d. vision

d. vision

Loundes Enterprises loads its products into truck trailers, then loads the trailers onto ships to transport long distances. Once the ship reaches the port nearest the final destination, the trailers are unloaded and hauled to their final destination by truck. Loundes' practice of transporting truck trailers on ships is known as ____________ shipping.

fishyback

Geographic segmentation.

segmenting markets by region of a country or the world, market size, market density, or climate

these are socially conscious groups that make it their mission to measure the social responsibility levels of businesses, and provide consumers with their opinions about the level of corporate responsibility of various companies.

1. An important source of public scrutiny is "watchdogs". These are: the efforts of a firm's competitors, including how often the competitors attack the firm's strategies. tax payer supported government agencies that limit a firm's ability to compete foreign markets. these are companies designated by the government to only produce products that support the government defense program. these are socially conscious groups that make it their mission to measure the social responsibility levels of businesses, and provide consumers with their opinions about the level of corporate responsibility of various companies.

on what social responsibility involves.

1. Business is under pressure from society to be more socially responsible. However, even the strongest advocates for corporate responsibility can't seem to agree: on what social responsibility involves. who should be policing business' behavior. if all businesses, large and small, should be held to the same standards. if politicians should be held to the same standards of behavior.

Which of the following is true of ethical decisions? a. Decisions made by organizations rather than individuals have ethical implications. b. Organizations do not influence the quality of an individual's decisions. c. When two people following an ethical framework are faced with choices, they always arrive at the same decision. d. Personal empathy in ethical choices in an important predictor of ethical leadership.

? quiz 3, answer isn't A

Which of the following statements is true of ethics? a. All ethical standards are codified in the law. b. Ethics and social responsibility are independent of each other. c. Establishing broad agreement on ethical standards is easier when a society is diverse. d. All actions that are legal need not be ethical.

? quiz 3, answer isn't C

Ethan is in the process of launching his new business venture that specializes in electronic waste recycling. During this period, he is in constant touch with Lloyd, who has seen success in this niche market. Which of the following entrepreneurial development strategies is Ethan utilizing? a. Gaining theoretical knowledge b. accessing Small Business Administration (SBA) resources c. Gaining experience d. Learning from others

? quiz 4, answer isn't B

Which of the following firms supported by the Small Business Administration (SBA) provides free, comprehensive counseling for small businesses from qualified volunteers? a. an angel investor b. the Service Corps of Retired Executives c. a Small Business Development Center d. a venture capital firm

? quiz 4, answer isn't C

Matt is asked by Clint Publishers to carry out an external audit of its books. He is also asked to prepare the tax returns for another client, Mrs. Zhirkob. Matt is a __________. a. public accountant b. management accountant c. government accountant d. cost accountant

? quiz 5, answer isn't B

Which of the following is a characteristic of debt financing? a. The requirement to make fixed payments. b. Restrictive covenant on borrowings. c. Higher risk than equity financing. d. Tax-deductive interest payments.

? quiz 6

A specialty store: a. offers a small range of everyday and impulse products at easy-to-access locations b. offers a wide range of food products, plus limited nonfood items c. sells a wide selection of merchandise within a narrow category. d. sells a complete selection of food and general merchandise at a steep discount in a single enormous location

? quiz 7, answer isn't A

market

A ________ refers to people with unsatisfied wants and needs who are willing and able to buy a product or service. market consumer cooperative consumer collective buyer's cooperative

business plan

A __________ is a written document that details the business idea, the target market and the business's competitive advantage, financial resources available for the business, and the qualification of the management. feasibility product marketing plan business plan statement of intent

line

A ____________ organization is characterized by direct two-way lines of responsibility, authority, and communication running from the top to the bottom of the organization, with all people reporting to only one supervisor.

vertical

A ______________ merger unites firms at different stages of related businesses. vertical horizontal diagonal conglomerate

demand

A __________________ curve shows the relationship between the price of a good and the quantity of that good people are willing and able to buy in a given time period.

Gantt chart

A bar graph that shows what projects or tasks are in process and how much of each is completed is called a(n): Grid chart. Gantt chart. Priority chart. Stem chart.

concerned with broad, long-term accomplishments, while objectives focus on specific short-term details.

A basic difference between goals and objectives is that goals are:

it permits customization without a significant increase in price.

A benefit of CAD (Computer Aided Design) and CAM (Computer Assisted Manufacturing) is: it basically eliminates the need for human intervention. it saves on materials costs. it permits customization without a significant increase in price. it produces a higher quality product which allows the manufacturer to charge more.

Funding a program of social responsibility can come from

A corporate that absorb the cost as a business expense

business plan

A detailed written statement that describes the nature of the business, the target market, the firm's competitive advantages, as well as the owner's resources and qualifications is business plan. entrepreneurial mission statement. profit and loss statement. professional action plan.

Soverign wealth funds

A fast-growing form of foreign direct investment is ______________ which are government controlled investments in foreign companies. sovereign wealth funds multinational wealth funds government mutual funds government wealth management derivatives

Provide people with better education

A key to economic growth and development is to: Locate more deposits of gold and other precious metals. Emphasize a fair and equitable distribution of income. Provide people with better education. Allow government planning to guide the allocation of resources.

communicate a vision and rally others around that vision.

A key to effective leadership is the ability to: maintain order, stability, and control. communicate a vision and rally others around that vision. master the technical details of the jobs performed by subordinates. maintain an autocratic approach to dealing with others.

Avoid the problem of double taxation associated with conventional corporations.

A major advantage of S corporations is that they: Can have more stockholders than a C corporation. Can operate in foreign nations as if they were domestic corporations. Require less paperwork to set up than a C corporation does. Avoid the problem of double taxation associated with conventional corporations.

loss of jobs in the US economy

A major concern voiced by U.S. critics of the North American Free Trade Agreement (NAFTA) is that it would result in: higher prices for consumer goods. loss of jobs in the U.S. economy. national security problems. increased illegal immigration from Mexico.

people must have unsatisfied wants and needs as well as a willingness and ability to buy.

A market consists of potential buyers of a good or service. However, for a market to exist: the business requires wholesalers and retailers. one firm must dominate in sales and product development. people must have unsatisfied wants and needs as well as a willingness and ability to buy. the product must be laboratory tested and approved by the Food and Drug Administration (FDA).

Taxed like a partnership

A master limited partnership (MLP) is: Not traded on the stock exchanges. Pays corporate income taxes. Taxed like a partnership. The corporate form of choice for small groups of individuals.

Conglomerate merger

A merger involving a software producer and a clothing manufacturer is an example of a: Vertical merger. Horizontal merger. Linear merger. Conglomerate merger.

Franchisee

A person who buys the right to use a business name and sell a product within a given territory is called a: Stockholder. Franchisee. Limited franchisor. Venture capitalist.

the movement toward flatter organizational structures, provide fewer opportunities to move upward.

A reason workers tend to receive fewer promotions than in the past is that: most firms now believe hiring people outside the organization is a better way to fill management positions because it brings in "new blood." the movement toward flatter organizational structures, provide fewer opportunities to move upward. firms have found that promoting employees can create jealousy and resentment among the workers who are not promoted. very few of today's supervisors and lower-level managers have the qualifications needed to move into higher management positions.

Overwhelming time commitment often required of the owner.

A significant disadvantage of owning a sole proprietorship is the: Possibility of limited liability. Heavy tax liability that must be assumed. Overwhelming time commitment often required of the owner. Lack of incentives to motivate the owner.

Price of a good and the quantity of that good sellers are willing to offer for sale at a specific point in time.

A simple supply curve shows a relationship between the: Amount of labor a firm hires and the amount of output it can produce. Amount of time required to produce a good and the relevant production costs. Price of a good and the quantity of that good sellers are willing to offer for sale at a specific point in time. Amount of a good a firm produces and the amount demanded by consumers.

Increase

A simple supply curve shows that an increase in the price of a good will cause the quantity supplied to: Decrease. Increase. Remain constant. Fluctuate randomly around its equilibrium value.

meet with experienced entrepreneurs and learn from their advice.

A small business management course at a community college may offer entrepreneurs an opportunity to: meet with experienced entrepreneurs and learn from their advice. try out ideas on a computer simulation before risking real time and money. listen to a qualified instructor solve all of the problems of small business owners. be part of a group that will include angel investors.

knockoff

A street vendor selling Rolex watches at ridiculously low prices approached Ted. Ted, however, was suspicious that the watches were illegal ________ brands. knockoff private generic look alike

ability to provide feedback to employees.

A strength of Generation X managers is likely to be their: loyalty to the firm. individualistic approach to problem solving. ability to provide feedback to employees. willingness to sacrifice other aspects of their lives to achieve success in the workplace.

Trademark

A symbol or a word that is registered with the US patent and trademark office and is legally protected from the use by anyone but its owner

Capital budget

A tool used by managers to estimate major expenditures for assets, expansion of facilities and mergers and acquisition

An employee's decision to inform the press or government officials of his or her firm's unethical practices is referred to as

A whistleblower

conducting a survey.

A widely used technique commonly used for collecting primary data is: collecting relevant articles in trade journals. benchmarking. conducting a survey. reviewing government reports.

assembly process

A(n) __________ puts together components to make a product. modular process compression process assembly process econometric process

operational plan

A(n) __________ would be used by a department manager as a tool for making daily and weekly assignments: operational plan vision statement mission statement tactical plan

organization chart

A(n) ________________ is a visual device which shows relationships among people and divides the organization's work.

conventional corporation

A(n) ___________________ is a state-chartered legal entity with authority to act and to have liability separate from its owners. limited partnership conventional corporation unlimited partnership nonprofit organization

another, higher order need would emerge to motivate that person.

Abraham Maslow thought that once needs at one level of his hierarchy were met: motivation would diminish. another, higher order need would emerge to motivate that person. that type of need would never again be an effective motivator. he or she would become more interested in lower-level needs.

Making the work itself more interesting and challenging

According to Herzberg's views, which of the following strategies would be most likely to result in a higher level of worker motivation? Improvements in the physical working conditions Providing clear and fair work rules and policies Making the work itself more interesting and challenging Using time-motion studies to find the most efficient techniques for employees to use

overall, motivators related to greater job satisfaction, while hygiene factors made the work place a satisfying place to thrive.

According to Herzberg, the difference between motivators and hygiene factors is: motivators cause persons to work smarter, while hygiene factors cause people to work harder. hygiene factors make employees more enthusiastic about the work itself, whereas motivators make employees enthusiastic about how many hours they have to work and their next day off. hygiene factors develop more loyalty from workers, while motivators make workers enthusiastic about moving on to a different company. overall, motivators related to greater job satisfaction, while hygiene factors made the work place a satisfying place to thrive.

maturity

According to the product life cycle model, sales are expected to peak in the _________ stage. introduction maturity growth commercialization

the formal organization and the informal organization.

All organizations have two organizational systems: the command system and the control system. the formal organization and the informal organization. the functioning system and backup system. the primary organization and secondary organization.

Collaborative Learning System it is a work environment that

Allows problem solving participation by team members

the EEOC (Equal Employment Opportunity Commission).

An important U.S. government organization charged with setting human resource management guidelines is: the EEOC (Equal Employment Opportunity Commission). the OSHA (Occupational Safety and Health Administration). the WPA (Work Progress Administration). the MPDA (Manpower Development Administration).

Imports exceed the value of exports

An unfavorable balance of trade occurs when the value of: Imports equal the value of exports. Imports exceed the value of exports. Cash inflows are equal to the value of cash outflows. Exports exceed the value of imports.

Balance Sheet

Another name for statement of financial position

Service Sector

Appliance repair firms, tax consultants, law firms, and insurance companies are all business firms that are part of the: production sector. management sector. neo-industrial sector. service sector.

division of labor.

As Brett prepares to open his new business, he has identified the tasks that need to be accomplished and has assigned employees to each task. This illustrates:

must become more like coaches and counselors than bosses.

As firms make greater use of empowerment and teams, managers will find that they: should use the same techniques for directing employees that were used in the past. will need to closely supervise the teams to make sure they remain focused on the goals of the organization. will need to provide the teams with detailed instructions to give them a clear sense of direction. must become more like coaches and counselors than bosses.

the long-term trend is for most nations to operate as mixed economies.

As the term "global" become "local", and as we see more opportunities to compete in the global market, the long-term trend is for most nations to operate as mixed economies. disagreements about the best economic system to embrace will cause many nations to close their doors to outside commerce. the only industries that governments will want to own are the ones that are profitable. the forecast is for socialist nations to trend toward communism (Cuba is the example) and for capitalist nations to trend toward socialism.

Accounts receivable and inventory

Assets most commonly used as collateral for short term financing

Tax rates on well-respected professionals such as doctors tend to be very high.

Brain drain has occurred in many socialistic nations because: Socialism encourages an unequal distribution of income, leaving many without medical and social services. The government does little to protect the natural environment. Workers tend to have to work longer hours and receive fewer benefits under socialism. Tax rates on well-respected professionals such as doctors tend to be very high.

Nadine has started up a new printing business. The printers, computers, and copying machines she has leased are examples of:

Capital

Internet services firms can use to recruit employees.

CareerBuilder and the Monster are examples of: web browsers. career interest surveys. Internet services firms can use to recruit employees. magazines that list job opportunities by skill requirements and geographic location.

hygiene factors.

Charlie, the CEO of Collier Chemical, likes to boast that his company offers the highest salaries in the industry, has excellent working conditions, and has clear and consistent company policies. Yet he admits his workforce is not highly motivated. Herzberg's research suggests that Collier Chemical is having difficulty motivating its employees because it focuses on: interactive factors. motivating factors. altruistic factors. hygiene factors.

social media

Companies are using _____________ to communicate their corporate social responsibility actions to their customers. sales promotion traditional advertising on television and radio social media their employees - word of mouth

benchmarking.

Companies benefit today from interfirm cooperation. One of the ways they cooperate is through networking. Another way is by seeking technological knowledge and learning new processes from exemplary firms sometimes outside their own industries. We call this:

ensuring equal pay for jobs requiring similar levels of skill, education and training.

Comparable worth primarily is concerned with: ensuring equal pay for men and women who work for the same firm and have the same job title. reducing the gap between pay for executives and the pay for non-managerial employees. ensuring equal pay for jobs requiring similar levels of skill, education and training. determining whether men or women are better suited to perform various jobs.

Ability to pool financial resources

Compared to a sole proprietorship, which of the following is considered an advantage of a general partnership? Ability to pool financial resources Unlimited liability for all owners Division of profits among owners Ease and flexibility in transferring shares of ownership to others

Has the ability to raise more money

Compared to partnerships and sole proprietorships, a major advantage of the C (conventional) corporation as a form of business ownership is that it: Has the ability to raise more money. Is easier and less expensive to form. Qualifies for simplified tax treatment. Creates unlimited liability for its owners.

can reach more destinations.

Compared to rail transportation, one of the advantages of shipping by truck is that trucks: are more energy efficient. can carry a greater volume. can reach more destinations. are less expensive.

have a more ambitious goal of not just satisfying customers, but of exceeding their expectations.

Comparing the business practices of the 1950s to those of today indicate that today's marketing managers: have a more ambitious goal of not just satisfying customers, but of exceeding their expectations. are much more focused on production and mass marketing. put less emphasis on earning a profit, since doing so makes it harder to satisfy the needs of the firm's stakeholders. operate essentially the same as their predecessors.

Payments that employees receive in return of their labor

Compensation

Non-price competition

Competition that is based on such as product quality, emotion, customer service and packing

vestibule training

Computer and robotics training is sometimes performed in a nearby classroom type setting. We refer to near-the-job-training as _____________. vestibule training orienteering apprenticeship training mentoring

The stage of new product development in which potential customers are presented with a written or oral description of the product to determine their initial attitudes and the buying intentions

Concept testing

ethical

Considering the potential impact on others prior to making a decision is consistent with ________ behavior.

cognitive dissonance.

Contacting customers after a sale to reassure them that they made the right decision is intended to offset: reverse satisfaction. cognitive dissonance. doubt resolution. relationship conflict.

establishing a currency that's tradable in world markets.

Contemporary governments promote business development by: increasing business regulations. providing government ownership of businesses. permitting businesspeople to conduct business without the use of contracts. establishing a currency that's tradable in world markets.

locate close together to decrease the cost of distribution.

Contemporary producers and their suppliers, locate in areas where they can share the same labor. know that it is not necessary to locate close to international customers due to technology such as the Internet. locate close together to decrease the cost of distribution. have advanced beyond ERP to more sophisticated systems.

mentors

Corporate managers who supervise, coach, and guide lower-level employees and serve as their organizational sponsors are called: networkers. role models. mentors. corporate guides.

____________ comprise about 20% of all businesses but account for about 81% of all business receipts.

Corporations

To determine if it is sensible for managers and employees to use the internet to solve problems firms use

Cost and benefit analysis

Franchising Disadvantages

Costs Lack of Control Negative Halo Effect Growth Challenges Restriction on Sale Poor Execution

fixed costs

Costs incurred regardless of the number of units of a product that are produced or sold are called: mixed costs. controllable costs. fixed costs. variable costs.

can encourage employees to deceive customers.

Creating competition between employees within the corporation: can encourage employees to deceive customers. should focus on improving corporate profit. must be ignored when corporate ethics are developed. can bring out the best in employees.

leadership is shared

Cross-functional self-managed teams work best when _______________. leadership is shared there are at least two line and staff mangers on the team the teams consist of members of only two functional units The academic levels of all members are comparable

A reduction in value of a nations currency relative to the curtains about the country is called

Currency devaluation

rail

Currently, in the United States the greatest volume of goods and services is shipped by: trucks and vans. pipelines. rail. water.

varies even among those who are interested in corporate responsibility.

Determining what is involved for a firm to be socially responsible: is easy. varies even among those who are interested in corporate responsibility. helps government officials develop appropriate legislation. varies from industry to industry due to different laws.

What indicates a customer orientation

Developing planets been satisfied customers needs

Working capital

Difference between Current assets and Current liabilities

low prices

Discount stores owe much of their success to a competitive strategy based on: low prices. extra service. special credit arrangements. a wide selection in a limited number of categories of goods.

maturity

During the ________ stage of the product life cycle sales reach a peak, profits are declining, and the number of competitors starts to decrease. growth maturity saturation decline

a member of the human resources department staff

During the employee selection process, the screening interview is often conducted by: the current workers in the applicant's department. a member of the human resources department staff. a selection committee consisting of managers from all departments within the organization. the company's affirmative action officer.

The organized effort of individuals to produce and sell for profit the products and services that satisfy society's needs through the internet

E-Business

monitors the computerized functions of all the divisions and subsidiaries of the firm in a single software program.

Enterprise resource planning (ERP) goes beyond the capabilities of MRP because it: gives much more precise cost estimates. finds the critical path and computes its length. monitors the computerized functions of all the divisions and subsidiaries of the firm in a single software program. handles research data about more than one product at the same time.

government programs that encourage economic growth through entrepreneurship.

Enterprise zones and investor visas are examples of: government policies that favor large corporations rather than small businesses. government programs that encourage economic growth through entrepreneurship. government policies designed to encourage intrapreneurs. examples of private initiatives to reduce the government's role in the economy.

accepting the risk of starting and running a business.

Entrepreneurship is: managing businesses for others. a controlling interest in the ownership of a corporation. managing businesses that operate in multiple countries. accepting the risk of starting and running a business.

If you want any manager who made sure that rewards are distributed to your employees fairly based on their performance and each employee clearly understood the basis for his or her own pay you would be using what theory

Equity Theory

people's perceptions

Equity judgments are based on: quantifiable evidence. deductive reasoning. statistical extrapolation. people's perceptions.

A computer-based system that facilitates and supports the decision making needs of top managers and Senior executives

Executive information

set attainable performance goals.

Expectancy theory suggests that managers attempting to improve employee performance would be well advised to: make sure that all employees are expected to meet the same standards. focus on setting monetary rewards for meeting the standards. set attainable performance goals. keep salaries earned by employees confidential.

secondary

Exploring _________ data first helps marketers minimize marketing research costs. focus group personal interview primary secondary

connect sellers in the U.S. with buyers in other nations, and help with customs, documentation and payments.

Export trading companies: enter into joint ventures with firms wanting to export goods and services. are government owned businesses that help companies go global. connect sellers in the U.S. with buyers in other nations, and help with customs, documentation and payments. is similar to a corporation that sells franchises.

customer relationship management

Firms that have adopted _________ seek to do more than simply sell a product; they seek to learn as much as possible about their customers in order to satisfy them with goods and services over time. nonprofit marketing customer relationship management management by objectives the marketing mix

nonprice competition

Firms utilize ________ when they attempt to add value to their product by offering service after the sale, product demonstrations, or interactive customer Web sites. nonprice competition relationship branding niche marketing break-even segmentation

can be helpful by offering them an opportunity to meet experienced small business owners.

For prospective entrepreneurs, enrolling in an introduction to business or small business management course offered through their community college: since college courses tend to be too theoretical. is likely to be wasted time since college courses tend to be too theoretical. can be helpful by offering them an opportunity to meet experienced small business owners. serves as supplementary information, only after starting their business venture. seldom provides you with information to help make small business decisions.

quality assurance

From the buyer's perspective, which of the following is a benefit of a product with a well known brand name? quality assurance helps in promotional efforts adds to repeat purchases differentiates products so that prices can be set higher

middle managment

General and divisional managers, plant managers, and deans and department heads at colleges are middle management. supervisory management. top management. nominal management.

A person who assumes full co-ownership of a partnership including unlimited liability is a

General partner

Types of Marketing Segmentation

Geographic segmentation. Demographic segmentation. Psychographic segmentation. Behavioral segmentation.

Which of the following is not one of the four resources used by businesses today

Governmental Do Use: 1. Material 2. Informational 3. Financial 4. Human

Businesses owned by minority women are growing at a faster rate than businesses owned by men or nonminority women.

How is the profile of business owners changing? In the U.S., men between the ages of 18 and 24 are starting businesses at a faster rate than men of all ages, or women of all ages. Recent male immigrants, who can legally work in the U.S., are starting businesses at a faster rate than natural-born male citizens, and at a faster rate than all women. Businesses owned by minority women are growing at a faster rate than businesses owned by men or nonminority women. It is difficult to collect information on the profile of business owners. Most new businesses do not file so they don't have to pay taxes.

process manufacturing

In ___________, materials are physically or chemically changed to produce a product. process manufacturing assembly processes utilitarian production diffusive manufacturing

matrix

In a _____________ organization, specialists from different parts of the organization are brought together on a temporary basis to work on specific projects. matrix line line-and-staff centralized

employees follow strict rules and regulations

In a bureaucratic organization: departments communicate with each other on a regular basis. employees follow strict rules and regulations. customer satisfaction is the number one priority. first-line workers are empowered to respond to the needs of customers.

employees are likely to have a sense of purpose and a common set of values.

In a company in which an effective vision has been provided: there is no need for tactical planning. the economic and competitive environments tend to be much more predictable. employees are likely to have a sense of purpose and a common set of values. goals and objectives are no longer needed.

Limited to losses that result from his/her own acts and omissions and the acts and omissions of those who work under his/her supervision.

In a limited liability partnership, each partner's risk of losing personal assets is: Unlimited. Limited to losses that result from his/her own acts and omissions and the acts and omissions of those who work under his/her supervision. Determined entirely by the maximum loss provision established by the articles of co-partnership. Nonexistent.

relationship building is considered a waste of time.

In a low-context culture: workers get right to the task, without the need to socialize during the first ½ hour of work. workers get right to the task, after they take care of their social needs. relationship building is considered a waste of time. relationship building is considered very important and occurs before performing business tasks.

workers must join the union within a stipulated time period (usually 30, 60, or 90 days) in order to keep their jobs.

In a union shop: workers must join the union within a stipulated time period (usually 30, 60, or 90 days) in order to keep their jobs. workers must belong to the union before the company can hire them. workers who do not join the union must pay a union fee. workers are required to sign yellow-dog contracts.

union

In a(n) __________ shop, workers do not have to belong to a union before they are hired, but they must join a union within a specified period (usually 30, 60, or 90 days) in order to keep their job. agency closed open union

apprenticeship program

In a(n) ______________ an inexperienced worker learns by working alongside an experienced employee to master the skills and procedures of a craft. orientation period simulation period indentureship program apprenticeship program

downliners

In multilevel marketing, independent contractors who sell a product can increase their own incomes by recruiting _________ to also sell the product. commissionaires front-men merchant wholesalers downliners

an asset that promotes harmony among workers.

In open organizations the informal organization can be: a hindrance to effective management. an asset that promotes harmony among workers. a roadblock to the corporate culture. part of the formal organization.

givebacks

In recent years some unions have granted concessions and given up benefits secured in earlier contract negotiations in an attempt to save jobs. These concessions are called: injunctions. givebacks. buyouts. disbenefits.

grown much more rapidly than the manufacturing sector.

In recent years, the service sector of the U.S. economy has: grown much more rapidly than the manufacturing sector. become the low-wage sector of the U.S. economy. achieved extremely high productivity gains according to government reports. focused less on quality and more on reducing costs of production.

cooperatives

In rural areas electrical power is often sold by ____________ that take advantage of the government's policy to sell them electricity at wholesale rates.

believe that management respected their ideas, which motivated them.

In the Hawthorne studies, employees were involved in the planning of the experiments. This caused employees to: set rather simple, easily attainable goals. become distracted and thus reduced productivity. believe that management respected their ideas, which motivated them. have high levels of productivity, but show disrespect for the managers in their test area.

closed shop.

In the late 1930s management at Bodenger Industries agreed to hire only those workers who were already members of the Steelworkers Union. Bodenger had agreed to a type of arrangement known as a(n): closed shop. open shop. union shop. restricted shop.

make large quantities of a limited variety of products at very low cost.

In the past, the idea behind mass production was to: produce goods of the highest possible quality. give producers the maximum flexibility to respond to customer preferences. make large quantities of a limited variety of products at very low cost. give workers more control over their work environment.

upliners are salespeople who act as independent contractors, earning commissions on their own sales and also on the sales of downliners, who are additional salespeople that they recruit.

In the terminology of multilevel marketing, which of the following statements about the relationship between upliners and downliners is most accurate? In multilevel marketing, upliners are intermediaries who deal directly with final customers, while downliners are intermediaries who coordinate the transportation and storage services the upliners need to move the goods they sell through the channel of distribution. upliners are salespeople who act as independent contractors, earning commissions on their own sales and also on the sales of downliners, who are additional salespeople that they recruit. upliners sell through electronic retailing arrangements, while downliners use more traditional methods of retail selling. upliners use intensive distribution strategies to sell a good, while downliners use either a selective or exclusive distribution strategy to sell the same type of good.

directing

In traditional organizations, ____________ involves giving explicit instructions to workers, telling them what to do to meet the goals and objectives of the organization.

technological advances such as the Internet and PayPal for safely fulfilling orders

Increased participation in small business exporting owes credit to ________. political constituents who have made in-roads abroad large events such as the Olympics that have brought attention to the potential in world markets technological advances such as the Internet and PayPal for safely fulfilling orders partnerships that small businesses have formed with large businesses

__________ refers to the strategy of using borrowed funds to increase the rate of return for stockholders.

Leverage

Disadvantages of Sole Proprietorships

Limited Financial Resources Unlimited Liability Limited ability to attract and maintain talented employees Lack of Permanence

Advantages of Corporations

Limited Liability Permanence Easy to Transfer Ownership Ability to Raise Capital Specialized Management

cash-and-carry wholesalers.

Limited-function wholesalers that mainly serve small businesses by selling them a limited assortment of goods are known as: backdoor wholesalers. consolidated wholesalers. cash-and-carry wholesalers. agency wholesalers.

a relatively unchanging environment.

Line-and-staff organizations work well in organizations with: operations in high-growth industries. stiff competition. short product life cycles. a relatively unchanging environment.

with flows of materials, finished products, and information.

Logistics is concerned: with flows of materials, finished products, and information. only with the flow of information, not the flow of materials and products. with the flow of materials to the manufacturer and with the flow of finished products to the consumer, but not with the flow of information. only with the flow of finished goods from the producer to the final consumer.

kiosks

Mall owners like to have __________ located along the walkways of their malls, because they create a marketplace type of atmosphere. cartels storettes kiosks minimarts

working in teams

Management is more progressive today. There is more emphasis on: individual work that can be successfully measured. working in teams. striving to eliminate the control function of management. placing greater reliance on the government for financing.

Franchising is a what kind of method

Management oriented method

it might be ok

Many Americans define ethical behavior according to the situation in which they find themselves. This suggests that there may be situations where ________ to cheat, steal, or lie. it is absolutely wrong it might be OK business people are expected capitalism forces people

The process of dividing a market into segments is called

Market segmentation

time

Marketing intermediaries add ________ utility to products by having them available when consumers want them. time allocative response form

possession

Marketing intermediaries can provide _________ utility by offering customers affordable credit, free delivery, and product guarantees. possession information form promotion

add value that exceeds the cost of their services.

Marketing intermediaries: add value that exceeds the cost of their services. add cost that exceeds the value they provide. increase the number of exchange relationships in the channel. complicate the distribution process.

intermittent processes.

Most new manufacturing facilities use: continuous processes. intermittent processes. traditional processes. synthetic processes.

tailored to the individual.

Motivational formulas are: tailored to the individual. tailored to the team of employees. developed by using one theory at a time. developed by using collecting historical information on the types of factors that motivated the majority of employees in the past.

Product differentiation

On its website, Papa John's Pizza compares itself to the competition with the following message: "better ingredients, better pizza". The company wants to achieve: Product differentiation. An equilibrium price. A monopoly. A perfectly competitive market.

Training method in which the training learned by working with a experienced employee

On-the-job training

it can return to an earlier stage through creative marketing.

Once a good or service nears the end of its product life cycle, the firm recognizes that: it is impossible to revert to an earlier stage of the model. it can return to an earlier stage through creative marketing. it will enjoy relatively high profits. marketing dollars spent now will provide little benefit.

innovation for new products occurs which keeps firms competitively challenged.

One advantage of free trade is: jobs are shifted to low-wage global markets. service jobs are moved overseas. advanced production operations are built in low-wage countries. innovation for new products occurs which keeps firms competitively challenged.

corporations

One disadvantage of _________ is the initial cost of formation. corporations general partnerships sole proprietorships limited partnerships

fear of negative publicity.

One factor causing businesses to vigorously enforce ethical standards is the: desire to eliminate employee turnover. fear of negative publicity. focus on criminal penalties. evasion of social audit measurements.

culture

One factor that influences the consumer decision-making process is _________, which refers to the set of values, attitudes, and ways of doing things passed from one generation to another in a given society. cognitive dissonance the legal environment group behavior culture

set through a process that involves all persons responsible for formulating and implementing the goals.

One implication of goal-setting theory is that goals should be: set by top management. relatively easy to achieve. set at a level that slightly exceeds the ability of employees to achieve them. set through a process that involves all persons responsible for formulating and implementing the goals.

promote conditions of fair competition for the three trading partners.

One objective of NAFTA is: increase trade barriers between the countries involved. decrease investment opportunities in the countries involved. promote conditions of fair competition for the three trading partners. improve working conditions in Central America.

Small Business Development Center.

One of the best places for young entrepreneurs to start shopping for a SBA loan is a: Small Business Development Center. financial institution specializing in business loans. college or university financial aid office. the Internet webpage: of the Venture Capitalists Association.

financing is often difficult to find

One of the hurdles small businesses face when deciding to export is: financing is often difficult to find. overseas buyers prefer dealing with large corporations. smaller companies must wait in line after large companies to qualify for tariffs. American products are perceived as inferior.

Unlimited liability the owner has for the debts of the firm

One of the major disadvantages of a sole proprietorship is the: Possibility of disagreements between owners. Unlimited liability the owner has for the debts of the firm. Fact that any income earned by this type of business is taxed twice. High cost of starting or ending the company.

a nationally recognized name and product.

One reason franchises have become so popular is that this arrangement provides the franchisee with:

find ways to appeal to white-collar, female, and foreign-born workers.

One strategy unions must adapt in order to grow in the future is to: find ways to appeal to white-collar, female, and foreign-born workers. find ways to prevent foreign companies from entering the U.S. market. go back to the old methods of aggressive tactics and confrontation that were used successfully in the past. emphasize recruiting efforts in the manufacturing sector of the economy.

May require the owner to adapt to social and cultural differences.

Opening and operating a franchise in a different country: Is illegal according to the Clayton Antitrust Act. Is no different than setting up a franchise in the domestic market. May require the owner to adapt to social and cultural differences. Is much less risky than owning a domestically based franchise.

New employees at Disney World in Orlando spend their first several days learning what it means to be a cast member by studying the traditions up Disney. This is called

Orientation process

industrial

Products that are used in the production of other goods and services are called ________ goods. specialty shopping industrial sponsored

The process of acquiring materials, supplies, components, and parts from another firm is known as

Purchasing

A market situation in which there are many buyers and sellers of a product and no single buyer or seller can control the price

Pure competition

Illegal behavior

Purposefully understating your firm's income to avoid paying higher taxes is an example of: An accepted business practice. Tax accountants doing their job. Illegal behavior. A firm's responsibility to its owners.

Businesses are generally placed into three broad categories

Services industry Production industry Distribution industry

on-the-job training

Shadowing is another term used to refer to _____________. job rotation vestibule training role playing on-the-job training

A company May extend the life of a product by

Significantly improving the products formulation

often find more job satisfaction than their counterparts in big business.

Small business employees: usually dislike their jobs and look forward to finding employment with major corporations. find their jobs less challenging than a similar job in a large corporation. usually get paid higher wages than workers in large corporations. often find more job satisfaction than their counterparts in big business.

Many jobs in small firms have low pay and few benefits

Small business owners often struggle in attracting qualified workers. Which of the following contributes to this problem? Bosses in small firms tend to be disrespectful to the workers Owners will not often listen to ideas suggested by the workers Employee relations are especially poor in small firms Many jobs in small firms have low pay and few benefits

individualized customer service as opposed to impersonal corporate bureaucracies.

Small businesses often have advantages in international markets. One advantage is: a greater expertise in managing the paperwork required to sell goods in international markets. a better understanding of the cultural differences involved. individualized customer service as opposed to impersonal corporate bureaucracies. the ability to obtain financing through the International Monetary Fund.

What is the Cornerstone of franchising

Standardization

believe in themselves and their ideas.

Successful entrepreneurs are self-nurturing. This means they: look to others for strength and encouragement. self-finance their business with no assistance from others. perform all business operations by themselves. believe in themselves and their ideas.

as a semi-private organization not under the direct control of the government.

The Federal Reserve Bank operates: as a semi-private organization not under the direct control of the government. under the direct control of the executive branch of the government. under the direction of the Banking Oversight Committee of Congress. as a special agency of the U.S. Treasury.

is independently owned but not dominant in its industry.

The Small Business Administration defines a small business as a firm that:

is independently owned but not dominant in its industry.

The Small Business Administration defines a small business as a firm that: has fewer than 100 partners. is independently owned but not dominant in its industry. is dominant in its industry, but small in the number of workers it employs. has annual receipts of more than a specified amount depending upon the industry.

integrity and sound business idea

The Small Business Administration's microloan program awards loans to entrepreneurs based primarily on their: collateral. credit history. previous business success. integrity and sound business idea.

consumer price index

The _________ is a statistic used to compute the pace of inflation or deflation. consumer price index gross domestic product index of leading indicators index of inflationary trends

Landrum-Griffin Act

The __________ guaranteed the rights of individual union members when dealing with their union. Wagner Act Landrum-Griffin Act Norris-LaGuardia Act Taft-Hartley Act

marketing concept

The __________ is based on three elements: (1) a customer orientation, (2) a service orientation, and (3) a profit orientation. mercantile philosophy hierarchy of needs relationship approach to marketing marketing concept

sole proprietorship

The __________ is usually the easiest form of business to start and end.

B2B

The __________ market consists of individuals and organizations that want goods and services in order to produce goods and services or to sell, rent, or supply those goods to others. consumer B2B target super

Universal Product Codes

The __________ on packages were intended to improve inventory control for businesses. stars and stripes Universal Product Codes lightweight adhesives Unit Standard Inventory bars

autocratic

The __________ style of leadership is characterized by making managerial decisions without consulting others.

Landrum-Griffin Act

The ___________ guaranteed union members the right to nominate candidates for union office and participate in union meetings. Taft-Hartley Act Fair Labor Standards Act Norris-LaGuardia Act Landrum-Griffin Act

grapevine

The ___________ is the nerve center of the informal organization. organizational database Intranet grapevine company newsletter

Age Discrimination in Employment Act of 1967

The ____________ protects individuals over the age of 40 from discrimination based on age with respect to issues such as hiring, promotions, job benefits, and assignments. Age Discrimination in Employment Act of 1967 Equal Employment Opportunity Act of 1998 Fair Labor Standards Act of 1938 Employee Rights and Responsibilities Act of 1983

National Labor Relations Board

The ______________ was established by the Wagner Act to oversee labor-management relations. Federal Trade Commission Commission on Labor Relations National Labor Relations Board Equal Employment Opportunity Commission

consumer price index

The ______________________ is a statistic used to compute the pace of inflation or deflation

Frederick Taylor research led to the emergence of Scientific Management which is

The application of scientific principles to the management of the work and workers

they protect domestic jobs by encouraging cost sensitive consumers to purchase domestic products.

The benefits of protective tariffs are: they increase the revenues of the government that has implemented them. they decrease the cost of the product to the consumer (end-user). they help develop friendlier trade relations with loyal trading partners. Both parties are protected. they protect domestic jobs by encouraging cost sensitive consumers to purchase domestic products.

honesty, integrity, and high ethical business standards.

The capitalist system relies heavily on: protectionist policies with respect to world markets. minimizing government intervention in corrupt business practices. making sure that currencies are never under-valued. honesty, integrity, and high ethical business standards.

variable

The cost of raw materials used to produce a good or service represent the firm's ________ costs. fixed variable mixed uncontrollable

A merger is the joining of resources of two companies, whereas an acquisition is a buyout of one firm by the other. The new company concerns itself with merging of resources.

The difference between a merger and an acquisition is: A merger does not combine the assets and liabilities of firms, whereas an acquisition combines assets and liabilities. A merger combines the assets of the two firms, but each company continues to assume its own liabilities, whereas an acquisition is a total buyout of one firm by another. A merger is the joining of resources of two companies, whereas an acquisition is a buyout of one firm by the other. The new company concerns itself with merging of resources. A merger is always something smaller tagging onto something larger, like a merging lane onto an interstate, whereas an acquisition is two firms that are relatively the same size agreeing to continue as one - more like two major interstates that come together and travel as one for several miles.

packaging carries more of the promotional burden of the product.

The evaluation of packaging as a marketing function indicates that: the package is less important as the Internet provides consumers with easily obtained information. packaging carries more of the promotional burden of the product. consumers are influenced less by packaging. the primary purpose of packaging is to limit the product liability of businesses.

the end use of the product

The factor that determines whether a product is classified as a consumer product or an industrial product is: the end use of the product. the decision-making process used to purchase the good or service. who the buyer represents. whether the good is new or used.

Knights of Labor

The first national labor organization in the United States was the: Knights of Labor. Congress of Industrial Organizations. American Federation of Labor. United Farm Workers Union.

define the problem and determine the present situation.

The first step in the marketing research process is to: define the problem and determine the present situation. collect relevant data from primary and secondary sources. analyze the research data. decide upon the best ethical solution.

define the situation

The first step in the rational decision-making model is to: recruit team members to work on the problem. gather data. develop alternative responses. define the situation.

cyclical

The four basic types of unemployment are frictional, structural, seasonal, and: cyclical. procedural. industrial. traditional.

The most important element for prospective franchisees is to investigate

The franchisor

less important

The further up the managerial ladder a person moves, the __________ his or her original job skills become. more important less important Stronger more tactical

is far reaching and affects all the other business environments.

The global environment, is a new frontier. is far reaching and affects all the other business environments. is shrinking. has not affected the United States due to its strong gross domestic product.

create a feeling of camaraderie among employees and encourage teamwork.

The informal organization of a business is probably best suited to: give workers a clear idea of where they stand in the organization. provide helpful guidelines about how to handle routine problems. provide a mechanism that generates careful reasoned decisions about critical issues. create a feeling of camaraderie among employees and encourage teamwork.

workers who do not join the union must pay a fee or regular dues, while in an open shop workers who choose not to join the union do not have to pay any union fees or dues.

The key difference between an agency shop agreement and an open shop agreement is that in an agency shop: workers must join the union within a stipulated time period (usually 30, 60, or 90 days) in order to keep their jobs, but in an open shop the workers are not required to join the union. the union is restricted to a limited number of employees who perform specific types of jobs, but in an open shop membership in the union is available to all workers. workers who do not join the union must pay a fee or regular dues, while in an open shop workers who choose not to join the union do not have to pay any union fees or dues. workers must agree not to join a union in order to keep their jobs while in an open shop workers are free to join a union if they wish, but they are not required to do so.

Foreign Corrupt Practices Act

The legislation that prohibits U.S. businesses from making "questionable" or "dubious" payments to foreign officials, to secure business contracts is the ______________. Foreign Corrupt Practices Act Foreign Anti-trust Act Multinational Collusion Law Global Good Conduct Law

Which of the following assessments of electronic retailing is most accurate?

The line between electronic retailing and traditional retailing is blurring as traditional retailers go online.

the matrix model is a temporary arrangement, whereas the cross-functional team is a more permanent arrangement.

The main difference between a matrix-style organization model and a cross-functional team is: the matrix model has a limit on the number of functional units who send an expert to join the team, whereas the cross-functional team does not. the matrix model is a temporary arrangement, whereas the cross-functional team is a more permanent arrangement. the matrix model shares leadership responsibilities within the group, whereas the cross-functional team does not. the matrix model is a more permanent arrangement, whereas the cross-functional team is a temporary arrangement.

Increase the amount of resources and create conditions that will make better use of those resources.

The main goal of resource development is to find ways to: Increase the amount of resources and create conditions that will make better use of those resources. Allocate existing resources more efficiently among competing uses. Find the right balance between policies that promote economic growth and policies designed to promote other goals such as a fair distribution of income. Promote the central planning of resource utilization so that resources can be allocated to the uses that will benefit society the most.

Personal selling

The most expensive but most adaptable method of the promotional mix

growth of consumer databases, blogs, and social networking.

The most important technological change likely to affect the marketing success of many firms is the: growth of consumer databases, blogs, and social networking. speed of service from faster check-outs. level of computer literacy of the target market. number of computers in use in the home.

Restaurants

The most popular type of business for franchising is: Consumer wholesale firms. Restaurants. Specialty steel manufacturing. Medical services.

Unlimited liability means

The owner is responsible for all the business debt

job rotation

The practice of moving employees from one job to another to make work more interesting is called: task mobility. job rotation. job incrementation. worker alternation.

late 1700s

The presence of formal labor organizations in the United States dates back to the: late 1700s. Civil War. late 1800s. depression years of the 1930s.

Supply of that product or service

The quantity of a good or service that producers are willing to sell at different prices during a specific period is known as the _______________________________

demography

The statistical study of human population to learn its size, density, and characteristics is called: psychographics. econometrics. demography. geometry.

Hawthorne effect.

The tendency for people to behave differently when they know they are being studied is known as the: Hawthorne effect. Taylor effect. Type II effect. induced motivation effect.

is costly to most firms because it is costly to governments who increase taxes to pay for added protection

The threat of terrorism, benefits most businesses because just about all companies contribute toward averting these threats. will certainly affect companies that participate in global trade, but most large U.S. companies do their business domestically. has increased the quality of life of most people because companies employ more workers when the threat is high, and workers help companies increase profits. is costly to most firms because it is costly to governments who increase taxes to pay for added protection.

Canada, Mexico, and the United States

The three countries participating in NAFTA are: Canada, Panama, and the United States. Mexico, Panama, and the United States. Canada, Nicaragua, and Mexico. Canada, Mexico, and the United States.

revenue and protective

The two basic types of tariffs are: goods and services. general and limited. comparative and absolute. revenue and protective.

storage and distribution.

The two basic types of warehouses are: public and private. railside and portside. storage and distribution. seasonal and long-term.

skill-based pay and gain-sharing.

The two most common compensation methods for teams are: cash bonuses and cashless bonuses. skill-based pay and gain-sharing. piecework pay and commissions. salaries and capital gains.

Which of the following best describes the opportunities available to small businesses in international markets?

The world is a much larger, and potentially more lucrative, market than the U.S. domestic market.

the average person dislikes work and will seek to avoid it when possible.

Theory X managers are likely to believe that: the average person dislikes work and will seek to avoid it when possible. most employees know more about their job than the boss. employees are motivated mainly by the chance for advancement and recognition. job satisfaction is primarily related to higher order needs.

What is the common mistake that small business owners make when they're business starts growing

They over-expanding without proper planning

The chance to be their own boss

To many businesspeople, one of the major attractions of a sole proprietorship is: The ability to obtain additional financial resources. The protection of limited liability. An unlimited lifespan. The chance to be their own boss.

Local businesspeople in the host countries.

To understand legal and regulatory conditions and be successful in global markets it's often useful to contact: The local Better Business Bureau. Local businesspeople in the host countries. The Regional Economic Development Trading Desk (REDTD). The Organization for Economic Cooperation and Development (OECD).

Traditionally small business statistics classified the retirement of an owner of a sole proprietorship as a business failure.

True

A country has an absolute advantage if it has a monopoly in the production of a specific product or is able to produce it more efficiently than all other nations.

True Page 62

entrepreneurship and knowledge

Two factors of production that seem to contribute the most to a nation's ability to create wealth are: land and labor. land and capital. entrepreneurship and labor. entrepreneurship and knowledge.

The goal of basic research is to

Uncover new knowledge without regard for its potential use

agency shop agreements

Under _______, workers are not required to join the union, but those who do not join are still required to pay a union fee or regular dues to the union. right-to-work laws all union security clauses agency shop agreements Employee Stock Option Plans (ESOPs)

Socialist

Under the __________ system the number of choices in the market is directly related to government involvement in markets. Capitalist Socialist Communist mixed economy

In the late 1930s management at Bodenger Industries agreed to hire only those workers who were already members of the Steelworkers Union. Bodenger had agreed to a type of arrangement known as a(n):

Union Shop

to seek protection against unfair treatment

Unions were originally formed _________________. to make certain that there were an equal number of nonmanagement and management-level employees within the company to uphold the state and federal laws under which businesses operate to seek protection against unfair treatment to seek worker corporate voting rights, the same as stockholders

Brand

Unique design, sign, symbol, words, or a combination of these, employed in creating an image that identifies a product and differentiates it from its competitors

Marketing plan

What are you going to achieve the marketing concept

generic name.

When a brand becomes commonplace and identified with a category of goods rather than the unique product of a specific manufacturer, the brand may become a: generic name. trademark. universal product code. product liability issue.

Institutional advertising

When a company uses celebrity to enhance the company's image

identify the things it does well as an organization and the things it needs to improve.

When a firm makes use of the SWOT analysis, one of its objectives is to: identify the best employees to fill each position within the organization. set specific short-term performance standards for each department. evaluate the desirability of issuing stocks or bonds in the current financial climate. identify the things it does well as an organization and the things it needs to improve.

the specific responsibilities of the job

When creating a job description, the HR manager may include: the minimum qualifications for the position the number of unpaid leave days the employee can take while at this position the specific responsibilities of the job a prediction of the number of similar positions that the company will need during the next two to five years

recognize that, while financial measures are important and should be considered, the purpose of the firm is to satisfy both internal and external customers.

When establishing criteria to measure the success of a customer-oriented business, managers should: limit their attention to financial measures such as profit and return on investment since these are the only objective and quantifiable measures available. focus most of their attention on measures of employee satisfaction, since without good employees the firm will not be competitive. focus primarily on financial measures, but pay some attention to non-financial concerns. recognize that, while financial measures are important and should be considered, the purpose of the firm is to satisfy both internal and external customers.

reverse logistics.

When firms manage the return of goods from consumers either because they are defective or for recycling purposes, they are involved in: materials handling. inbound logistics. factory processes. reverse logistics.

foreign direct investment

When foreign firms build production facilities in the United States, they are engaging in: exporting. foreign direct investment. importing. countertrading.

They expect change

When it comes to motivating Millennials and Gen Xers, it is important for managers to know that they have a few things in common. One important likeness is: They don't like large families They expect change Neither group was raised by baby boomers Neither group has advanced to management positions

planning

When managers identify a market trend that suggests a new opportunity and then devise a strategy to go after this new opportunity, they are involved in the function of: controlling. planning. leading. organizing.

a surplus will exist in the market

When the market price of a good is above its equilibrium value, and all other determinants are unchanged: _________________________

Equilibrium price of the good

When the supply curve and demand curve for a particular good are shown together on a single graph, the point at which the two curves intersect identifies the: Total profit earned by producers. Total amount of labor that will be employed in that market. Amount of time it takes to bring together the buyers and sellers of the good. Equilibrium price of the good.

conglomerate

When two firms who do not participate in the same industries, for example a software company and a fast food restaurant company decide to merge, the result is called a ____________ merger. vertical horizontal linear conglomerate

wealthy nations have knowledge and entrepreneurial opportunities, while poor nations are often lacking in these areas.

When we compare the factors of production in wealthy and poor nations, we find: poor nations have plenty of land and knowledge, but very little labor. poor and wealthy nations (both) have an abundance of knowledge, it is the land that varies, with rich nations always having more. wealthy nations have knowledge and entrepreneurial opportunities, while poor nations are often lacking in these areas. wealthy nations have land and labor, while poor nations have capital and entrepreneurship.

Looking at market forecasts to identify future business opportunities and challenges

Which of the following activities is part of the planning function of management? Assigning a particular worker to do a specific task Looking at market forecasts to identify future business opportunities and challenges Praising a sales representative who has far exceeded her monthly sales objectives Conducting a job interview with a potential new employee

Intrapreneurs work within existing corporations while entrepreneurs assume the risks of starting their own business

Which of the following best describes the difference between an intrapreneur and an entrepreneur? Intrapreneurs start nonprofit organizations while entrepreneurs start profit-seeking businesses Intraprenuers are practical while entrepreneurs are creative Intrapreneurs work within existing corporations while entrepreneurs assume the risks of starting their own business Intrapreneurs limit their efforts to financial investments, while entrepreneurs are involved in the management of all areas of a business

the world is a larger, and potentially a more lucrative market than the U.S. domestic market

Which of the following best describes the outlook for small businesses in international markets? the world is a larger, and potentially a more lucrative market than the U.S. domestic market the threat of terrorist activities prevents small businesses from competing internationally most foreign buyers prefer to deal with large, well-known corporations rather than small businesses higher business costs eliminate the profit opportunities in global markets

Restructuring

Which of the following describes the redesigning of an organization so that it can more effectively and efficiently serve its customers? Reinventing Restructuring Radical reorganizing Extroverted reorganizing

(S)he has a capacity to serve as a counselor and an investigator.

Which of the following is a characteristic of an effective ethics officer? (S)he has a background in human resource management. (S)he is exceptionally organized. (S)he has a capacity to serve as a counselor and an investigator. (S)he communicates primarily via email.

action-oriented

Which of the following is a characteristic of successful entrepreneurs? Power-seeking Relies heavily on others who believe in the business Willingness and desire to follow others Action-oriented

Access to various modes of transportation

Which of the following is a likely reason that a company would move its facility from one location to another? Availability of expensive labor Higher quotas on competing goods Access to various modes of transportation Availability of a retired workforce

An increasing number of companies are hiring skilled professionals such as accountants as part of their contingent workforce.

Which of the following is a valid statement about contingent workers? Contingent workers are seldom skilled professionals. By law, employers must provide contingent workers the same salary and benefits as permanent employees. An increasing number of companies are hiring skilled professionals such as accountants as part of their contingent workforce. By law, companies cannot hire contingent workers as part of the management team.

Companies can create efficiencies by hiring low-wage employees.

Which of the following is an advantage of offshore outsourcing? Companies can create efficiencies by hiring low-wage employees. Wages increase in the home country. Product quality improves permitting firms to charge more for their products. Communication between the company and its customers improves.

Labor Standards

Which of the following is an unresolved ethical issue that surrounds global trade? Product quality Labor standards Transportation issues Time to market issues

defining the question and determining the present situation

Which of the following is considered a step in the marketing research process? designing a product to meet the need based on research defining the question and determining the present situation planning for product modifications and test advertising setting the price for a product

There may be a lack of communication among the different departments

Which of the following is considered to be a disadvantage of functional departmentalization? There may be a lack of communication among the different departments Employees must develop skills in depth The company must achieve economies of scale Top management has to direct and control department activities

millitary expenditures

Which of the following is considered when calculating a country's balance of payments? military expenditures state unemployment domestic inflation rates foreign inflation rates

working with buyers to determine their individual needs

Which of the following is consistent with relationship marketing? maintaining transactional relationships with customers working with buyers to determine their individual needs developing market share through mass media advertising focusing more on obtaining new customers than on retaining existing customers

Sexual harassment

Which of the following is likely to remain a controversial labor-management issue in the future? The ability of employers to use yellow-dog contracts Enforcement of the Taft-Hartley Act Sexual harassment A resurgence of the Knights of Labor

Kellogg offers Special K, Raisin Bran, Corn Flakes, and a variety of other cold ready-to-eat cereals.

Which of the following is the best example of a product line? General Motors offers passenger cars, small trucks, and tractor trailers. Sony offers DVD players, compact disk players, and video games. Kellogg offers Special K, Raisin Bran, Corn Flakes, and a variety of other cold ready-to-eat cereals. Hewlett Packard offers scanners, printers and personal computers.

Cross-functional teams work best when the voice of the customer is brought into the organization

Which of the following is true of cross-functional teams? Cross-functional teams are required not to share information across national boundaries Employees on the teams are frustrated by organizational rules and regulations Cross-functional teams work best when the voice of the customer is brought into the organization Cross-functional teams are disbanded once the current project is complete

An attorney in the legal department of a manufacturing corporation

Which of the following positions would be classified as a staff position? An assembly line worker for a major auto manufacturer The chief executive officer of a small corporation A sales manager for a chain of clothing stores An attorney in the legal department of a manufacturing corporation

provide the opportunity for exit interviews when employees leave or are discharged

Which of the following practices would fail to result in more open communication? provide opportunities such as company picnics and open lunch rooms that promote active communication among employees and management develop opportunities for employees to offer open-ended responses to internal surveys provide the opportunity for exit interviews when employees leave or are discharged provide training for improving listening skills

a limited-edition print signed by the artist

Which of the following products would most likely be classified as a specialty good or service? a microwave oven a limited-edition print signed by the artist a sports coat, slacks, and tie a car insurance policy

Is it balanced?

Which of the following questions will help individuals and organizations when deciding if their decisions are ethical?

Some organizations have policies that demand promotions from within that limits the pool of candidates.

Which of the following reasons makes the recruiting job more challenging? The current emphasis on scientific management requires very specific job specification. Some organizations have policies that demand promotions from within that limits the pool of candidates. Reverse discrimination is essential to be sure that the workforce is balanced. There are far too many qualified candidates for each position, increasing the required paperwork.

penetration strategy

Which of the following represents a pricing strategy that establishes a low price in hopes of attracting a great number of customers and attempts to discourage competitors? penetration strategy odd-even pricing strategy skimming strategy price lining strategy

An automobile manufacturer offers different types of cars designed to appeal to different age groups and income levels.

Which of the following represents an example of demographic segmentation? A firm producing recreational equipment divides its market into groups by region of the country, designing different products for people in different parts of the country. An automobile manufacturer offers different types of cars designed to appeal to different age groups and income levels. A motion picture company divides its market according to the attitudes and interests of its customers, marketing some films to people who enjoy action movies and targeting other movies to people who prefer romantic comedies. A company that produces food products has designed a special line of easy to prepare microwavable meals that are low fat and high in fiber to attract people interested in the health benefits of the foods they eat.

Manufacturers often seek to reduce time-to-market by locating facilities in areas with access to key modes of transportation

Which of the following statements about facility location is most accurate? Very few firms are still influenced by labor costs when making decisions about where to locate Recent changes in information technology have led firms to favor locations near major urban areas Manufacturers often seek to reduce time-to-market by locating facilities in areas with access to key modes of transportation No matter how large they become, most companies ultimately choose to locate major facilities within a few miles of the area in which they started business

Home-based work allows workers to choose their own hours and interrupt work for child care and other tasks, but this freedom and flexibility can create problems for some workers.

Which of the following statements about home-based work and telecommuting is most accurate? Although home-based work may become very popular in the future, only a small percentage of workers currently work at home on a regular basis. Telecommuting offers big advantages for many workers, but is likely to increase costs for employers significantly. Home-based work tends to increase the productivity of almost all workers, since people tend to be more productive when they are comfortable and can work flexible hours. Home-based work allows workers to choose their own hours and interrupt work for child care and other tasks, but this freedom and flexibility can create problems for some workers.

workers are becoming less loyal to their firms, resulting in higher turnover

Which of the following statements about human resource management opportunities and challenges is most accurate? the age composition of the labor force is becoming younger workers are becoming less loyal to their firms, resulting in higher turnover workers are now demanding that more of their compensation be in wages and salaries and less in fringe benefits the number of workers who have received training in high tech fields far exceeds the number of job openings in these areas

Students can begin building network connections in college, even before they take on a full time job.

Which of the following statements about networking is the most accurate? White males are much more aware of the importance of networking than women and minorities. Although African-American and Hispanic managers are becoming aware of the importance of networking, there are no organizations available to help them make networking connections. Women have more difficulty forming networks than men because they can be barred from male only clubs and social organizations where much of the networking occurs. Students can begin building network connections in college, even before they take on a full time job.

Industrial buyers are usually large.

Which of the following statements about the B2B market is true? The markets are often geographically dispersed. Industrial buyers are usually large. Industrial buyers tend to be more emotional in their purchasing decisions. The size of the market is smaller than the consumer market.

The costs of elder care are likely to rise very rapidly in coming years as an increasing number of older and more experienced employees face the need to care for aging parents and other relatives.

Which of the following statements about the costs of elder care is most accurate? It should be much less expensive for firms to provide elder care than for them to provide childcare. The financial burdens of elder care on the children of aging parents are likely to become less serious in the future than they are today, because many senior citizens are now remaining employed into their 70s rather than retiring in their early 60s. Although elder care is expensive, the good news is that companies can receive a great deal of assistance from the federal government when they establish qualified elder care programs. The costs of elder care are likely to rise very rapidly in coming years as an increasing number of older and more experienced employees face the need to care for aging parents and other relatives.

The same product can be classified as either a consumer good or an industrial good, depending upon its end use.

Which of the following statements about the distinction between consumer goods and B2B goods is the most accurate? The same product can be classified as either a consumer good or an industrial good, depending upon its end use. Industrial goods tend to have a useful life of several years and can be depreciated while consumer goods typically last less than a year. Consumer goods are typically more expensive than industrial goods. Industrial goods are custom made to the specifications of the buyer, while the all consumer goods are mass-produced.

A corporation receives its charter from a state government.

Which of the following statements about the operation of a corporation is correct? A corporation receives its charter from a state government. A corporate charter automatically expires in 99 years and must be renewed if the corporation wants to remain in business. Owners of a corporation have unlimited liability for any claims against their company. A corporation tends to be much easier to set up than a sole proprietorship or partnership.

are independently owned, but cooperate as a unified system of stores.

Which of the following statements about wholesaler-sponsored chains is most accurate? In a wholesaler-sponsored chain all stores: are independently owned, but cooperate as a unified system of stores. act independently except for an agreement to share the costs associated with distribution. are owned by a single wholesaler, but operate under separate management. share profits and losses according to a contractual arrangement.

Firms that offer real value will develop a competitive advantage and gain customer trust.

Which of the following statements best describes a firm's commitment to its customers?

Each nation should produce those goods that it can produce more efficiently and effectively than other nations, and buy the goods it cannot produce efficiently from other nations.

Which of the following statements best describes the theory of comparative advantage? The best way for a nation to ensure full employment is to be totally self-sufficient rather than relying on other nations to obtain goods and services. Each nation should produce those goods that it can produce more efficiently and effectively than other nations, and buy the goods it cannot produce efficiently from other nations. The nation that has the largest reserves of gold and other natural resources will enjoy a position of comparative advantage in trade relationships. A nation should produce those goods for which domestic demand is comparatively strong, and should import those goods for which domestic demand is comparatively weak.

No central system of law exists

Which of the following statements describes laws and regulations in global markets? No central system of law exists. The World Trade Organization reigns supreme and dictates law for all global markets. Most trading partners will default to the laws that govern the seller's home country. With the exception of labor relations, most laws are pretty standard across global markets.

Training is short-term skills oriented while development is long-term career oriented.

Which of the following statements describes the relationship between employee training and employee development? They are just two names for the same thing. Training is short-term skills oriented while development is long-term career oriented. Development is provided by the company and is designed to meet the specific needs of the organization, whereas training focuses on more general skills and normally is provided by schools, colleges and institutions other than the employer. Training focuses on conceptual skills while development seeks to improve technical skills.

A job description is a statement about the job, while a job specification is a statement about the person who does the job.

Which of the following statements identifies a key difference between a job description and a job specification? A job description is concerned with the short-run objectives of a job, but a job specification is concerned with the job's long-run objectives. A job description is based on a human resource inventory, while a job specification is the result of a performance appraisal. A job description is a statement about the job, while a job specification is a statement about the person who does the job. A job description is a brief summary of what a job involves, while a job specification is a detailed and technically complete statement of what a job involves.

Most manufacturing firms are turning to an assembly line layout to produce output as efficiently as possible

Which of the following statements is most accurate? Most manufacturing firms are turning to an assembly line layout to produce output as efficiently as possible For service firms, the main objective in facility layout usually is to centralize the control of operations Facility layout is important for manufacturing plants because facilities layout decisions can have a huge impact on the firm's costs In a modular layout, workers are assigned a specific station along an assembly line and specialize in performing a specific task

Employees are like machines that must be programmed to perform in a certain way

Which of the following statements is the best description of how scientific management viewed employees? Employees are like machines that must be programmed to perform in a certain way Most employees are creative and intelligent individuals who should be given a great deal of freedom and flexibility in how they perform their jobs Employees will only exert their maximum effort if they believe that their goals are attainable

Brand awareness and image are important in marketing these products.

Which of the following statements regarding convenience goods and services is accurate? Shoppers will exert lots of effort to obtain the product. Consumers purchase the product on an infrequent basis. Consumers buy only after carefully comparing quality and value. Brand awareness and image are important in marketing these products.

Workers should simply follow well-defined procedures

Which of the following statements would best summarize Max Weber's views on the role of workers in an organization? Workers should be empowered to do whatever it takes to satisfy the needs of customers Workers will be promoted within the organization depending upon "who they know", rather than "what they know" Workers should simply follow well-defined procedures Although top managers must ultimately make the decisions, workers are an important creative resource for a firm, and top management should listen to their ideas

advertise the opportunity for a promotion to current employees

Which of the following strategies is used by a human resource manager whose goal is to fill an open position from within the company? advertise the opportunity for a promotion to current employees seek the assistance of a public employment agencies send the job opening to Monster.com utilize the services of a private employment agency

just-in-time (JIT)

With the __________ inventory control system, producers hold only the amount of materials and parts that they need for a short period of time - sometimes just a few hours. ABC first-in, first-out just-in-time (JIT) maximum

online training programs

With the advent of more sophisticated technology, employers who operate locally and globally and want all employees to receive the same training may opt for _____. apprenticeship programs mentorship programs job rotation programs online training programs

Marketing

________ is the activity, set of institutions, and processes for creating, communicating, delivering, and exchanging offerings that have value for customers, clients, partners, and society at large. Selling Advertising Marketing Mixing markets

Inbound logistics

_________ brings raw materials, packaging, other goods and services, and information from suppliers to producers. Freight forwarding Inbound logistics Materials handling Supply side logistics

Logistics

_________ involves planning, implementing, and controlling the physical flow of materials, final goods, and related information from points of origin to points of consumption to meet customer requirements at a profit. Export management Multi-level marketing Logistics Shipping control management

Packaging

_________ is an important part of the total product offer for a product. Consumer income Management style Employee personality Packaging

Adam Smith

_________ is considered by some to be the father of modern economics. John Maynard Keynes Adam Smith Thomas Carlyle Thomas Malthus

Crisis

_________ planning is a part of contingency planning that is used to deal with sudden and unexpected changes in the business environment. Crisis Tactical Operational SWOT

line personnel

__________ are responsible for directly achieving organizational goals. Organizational specialists Line personnel Chain of command managers Staff personnel

Max Weber

__________ believed that managers were trustworthy and companies would do well if employees simply did what they were told. John Keynes Adam Smith Peter Drucker Max Weber

Specialty

__________ goods appeal to consumers who are willing to make an extra shopping effort to acquire them. Specialty Shopping Convenience Consumer

Target Costing

__________ involves designing a product so that it satisfies customers and meets the profit margins desired by the firm. Price discrimination Price leadership Target costing Cost-based pricing

Planning

__________ involves setting the organization's vision, goals, and objectives. Selected Answer: Organizing Planning Controlling Directing

Deflation

__________ is a condition in which the average level of prices is actually falling. Deflation Disinflation Stagflation Uniflation

Decision-making

__________ is choosing among two or more alternatives. Contingency planning Decision-making Organizational selection Utility analysis

Profit

__________ is the amount a business earns after deducting what it spends for salaries and other expenses. Profit Revenue Interest Dividends

Management

__________ is the art of utilizing organizational resources to accomplish goals at work. Socialization Justification Preparation Management

Production

__________ is the creation of goods and services using land, labor, capital, entrepreneurship, and knowledge. Resource recombination Supply chain management Production Purchasing

Bartering

__________ is the exchange of merchandise or services for other merchandise or services without the exchange of money. Exporting Bartering Importing Factoring

Facility layout

__________ is the physical arrangement of resources in the production process. Facility location Facility layout ERP RRL

Dumping

__________ is the practice of selling a product in foreign countries for a lower price than the good is sold in the producing country. Deflating Countertrading Inflating Dumping

Certification

__________ is the process by which a union becomes recognized by the National Labor Relations Board as the bargaining agent for a group of employees. Accreditation Certification Arbitration Affiliation

Facility location

__________ is the process of selecting a geographic location for a company's operations. Facility location Facility layout FRP Geographic requirements planning

Break-even analysis

__________ is the process used to determine the profitability of a product at various levels of sales. Cost-based pricing Break-even analysis Demand-based pricing Competition-based pricing

Strategic

__________ planning determines the major goals of an organization and lays the foundation for obtaining and using resources to achieve those goals.

Contingency

__________ planning prepares alternative courses of action that may be used if the primary plans are not achieving objectives. Strategic Contingency Tactical Alternative

Frictional

__________ unemployment refers to persons who purposefully quit their jobs and have not yet found a new job. Frictional Structural Cyclical Seasonal

Structural

__________ unemployment refers to unemployment caused by the restructuring of firms or by a mismatch between the skills of job seekers and the requirements of available jobs. Frictional Structural Cyclical Seasonal

Full-service wholesalers

___________ are merchant wholesalers who perform all of the distribution functions. Retailers Intensive contact wholesalers Total-capacity wholesalers Full-service wholesalers

Monopolistic competition

___________ exists when a large number of firms produce goods that are similar but are perceived by buyers as being different. Perfect competition Monopolistic competition Oligopoly A monopoly

Staffing

___________ involves recruiting, hiring, motivating, and retaining the best people available to accomplish the company's objectives. Directing Planning Leading Staffing

Enterprise resource planning (ERP)

___________ is a computer application that enables a firm to combine computerized functions of all divisions and subsidiaries of the firm into a single, integrated software program that uses a single database. Materials requirement planning (MRP) PERT analysis Manufacturing resource planning (MRP II) Enterprise resource planning (ERP)

Communism

___________ is a free-market economic system in which most of the factors of production and distribution (land, railroads, stores) are owned by individuals

PERT

___________ is a technique to analyze and sequence tasks involved in completing a project, to estimate the time needed to complete each task, and to determine the minimum amount of time needed to complete each project. PERT ECON LIFO GANTT

Leading

___________ is the management function of creating a vision for the organization and guiding, training, coaching, and motivating employees to help achieve the goals and objectives of the organization. A. Organizing

Planning

___________ is the management function that involves setting goals, strategies, and tactics for achieving the organization's goals and objectives. Forecasting Planning Conceptualizing Controlling

Management development

___________ is the process of training and educating employees to become good managers and then developing their managerial skills over time. Management development Vestibule training Pre-mentoring Self-actualization training

Mediation

___________ is the use of a third party to encourage labor and management to continue negotiating in an effort to settle a labor dispute or achieve a mutually acceptable labor-management agreement. Arbitration Mediation Reconciliation Intercession

Trade Protection

___________ is the use of government regulations to limit the import of goods and services. Trade protectionism Fiscal policy Countertrade policy Monetary policy

effiency

___________ means producing items using the least amount of resources. Efficiency Technology E-production Productivity

Macroeconomics

___________ studies the operation of a nation's economy as a whole. Microeconomics Socioeconomics Econometrics Macroeconomics

knowlege management

____________ is concerned with finding the right information and making that information accessible and understood by everyone in an organization. Mission management Data management Knowledge management Auditing

Environmental scanning

____________ is the process of identifying factors that can affect marketing success. Environmental scanning Target marketing Niche marketing Segmentation analysis

sexual harassment

____________ refers to unwelcome sexual advances, requests for sexual favors, and other unsolicited conduct of a sexual nature. Sexual harassment Gender bias Sexual opportunism Sexual coercion

Networking

_____________ is using communications technology and other means to link organizations and allow them to work together on common objectives.

Empowerment

_____________ means giving employees the authority and responsibility to respond quickly to customer requests. Restructuring Delineating Empowerment Intrapreneuring

Export trading companies

_______________ are specialists that match buyers and sellers from different countries and provide services to ease the process of entering global markets. Export trading companies Licensees Strategic alliances Keiretsu

The business enviorment

_______________ is/are the external and internal circumstances that surround the business. The business environment The factors of production The business corral The business disciplines

Macroeconomics

_______________ studies the economy as a whole

Physical and enviormental

_________________ differences such as insufficient electrical power and transportation systems can present special problems for U.S. firms when attempting to enter global markets. Physical and environmental Legal and regulatory Economic Governmental

Frederick Taylor

_________________ was the father of scientific management. Elton Mayo Frank Gilbreth Henry Gantt Frederick Taylor

Staffing

___________________ involves recruiting, hiring, motivating, and retaining the best people available to accomplish the company's objectives.

Which of the following statements is true of the impact of federal regulations on entrepreneurship? a. Extensive regulations can impeded entrepreneurship. b. Lack of regulations can stop entrepreneurship from thriving. c. Entrepreneurs owning small businesses in favor of strict regulations. d. Regulations and entrepreneurship are not interdependent.

a Extensive regulations can impede entrepreneurship.

Services industry

a business that does work for a customer, and occasionally provides goods, but is not involved in manufacturing.

______ refers to marketing, buying, selling, and servicing of products over a network, usually the internet. a. E-commerce b. Media-based networking c. Enterprise content management d. Social commerce

a. E-commerce

Which of the following statements is true of enterprise resource planning (ERP)? a. ERP makes it easier for organizations throughout a supply chain to coordinate their activities. b. ERP systems have a simple structure that is inexpensive and easy to implement c. ERP software requires users to enter and access data in traditional way d. ERP is the software version of the critical path method that is used for project management

a. ERP makes it easier for organizations throughout a supply chain to coordinate their activities.

In operations management, using resources to create value by providing customers with goods and services that offer a better relationship between price and perceived benefits is known as _____. a. Effectiveness b. Efficiency c. Throughput d. Productivity

a. Effectiveness

____ refers to the physical components used to collect, input, store, and process data, and to display and distribute information. a. Hardware b. Software c. Intranet d. Extranet

a. Hardware

During his tenure as the manager of a large toy store, Barry developed strong technical skills. He will soon be promoted to the position of regional manager for several branches of the toy store. Barry's boss informs him that in this new role, he will need to communicate, coach, and motivate other managers. In this case, which of the following management skills is his boss describing? a. Human skills b. Conceptual skills c. Technical skills d. Processing skills

a. Human skills

Which of the following statements is true about the markets around the world? a. India's subscriber base for cell phones has grown explosively over the past five years. b. China has lower gross domestic product (GDP) growth rate than the U.S. c. China and India have a smaller opportunity to attract investments because of their economic growth. d. Most of the developing countries remain ahead of the United States in terms of development and prosperity.

a. India's subscriber base for cell phones has grown explosively over the past five years.

Which of the following statements is true of a product layout? a. It organizes machinery, equipment, and other resources according to the specific sequence of operations that must be performed. b. In this layout, a product can be moved from one type of machinery to another in the specific sequence of operations that must be performed. c. It is used for goods that must be produced at a specific site or that are so large and bulky that it is not feasible to move them from station to station. d. It is used by firms that need to produce small batches of goods that require a degree of customization.

a. It organizes machinery, equipment, and other resources according to the specific sequence of operations that must be performed.

Which of the following statements best describes a money market mutual fund? a. It pools funds from many investors and uses these funds to purchase very safe, highly liquid securities. b. It is a short-term marketable IOU issued by the U.S. federal government c. It is the part of a firm's net income that it reinvests d. It is a financial agreement between a firm and a bank in which the bank pre-approves credit up to a specified limit.

a. It pools funds from many investors and uses these funds to purchase very safe, highly liquid securities.

In today's competitive markets, operations managers: a. Realize that a product that offers better features is more attractive than one that has a lower price. b. Recognize that achieving is more important than achieving effectiveness. c. Concentrate primarily on keeping costs low to keep prices competitive. d. Focus solely on the benefits of products, while completely ignoring the costs of those products.

a. Realize that a product that offers better features is more attractive than one that has a lower price.

Which of the following statements is true of robots? a. Robots perform jobs that human workers find dangerous or physically demanding. b. Robots' performance levels decrease when they have to work for very long hours. c. Robots are usually immobile and perform jobs statically. d. Robots are inflexible and cannot be reprogrammed to perform new tricks.

a. Robots perform jobs that human workers find dangerous or physically demanding.

______ refer to expertise in a specific functional area or department. a. Technical skills b. Human skills c. Conceptual skills d. Processing skills

a. Technical skills

Which of the following statements is true of necessity entrepreneurship in high-income countries? a. The rate of necessity entrepreneurship is low. b. The risks of necessity entrepreneurship are low. c. Necessity entrepreneurship is a major source of income. d. Necessity entrepreneurship is nonexistent.

a. The rate of necessity entrepreneurship is low.

Which of the following is true in the context of the steps that need to be taken by top managers in executing the integrated approach to the implementation of a code of ethics in an organization? a. They should ensure that the company's ethics code is both local and global in scope. b. They should ensure that they do not overcommunicate the importance of ethics. c. They should separate the ethics training given to employees from mandatory staff training. d. They should enforce strict laws and penalties against whistleblowers in the organization.

a. They should ensure that the company's ethics code is both local and global in scope.

Dividends to common stockholders of a company are: a. a distribution of earning that are paid only if declared by the company's board of directors b. a legally required payment to the company's shareholders c. a tax-deductive expense d. the result of a rise in the market price of the firm's stock in the secondary market

a. a distribution of earning that are paid only if declared by the company's board of directors

The _____ reports the financial position of a firm by identifying and reporting the value of the firm's assets, liabilities, and owners' equity. a. balance sheet b. profit and loss statement c. statement of retained earnings d. statement of cash flows

a. balance sheet

The economic system of the United States is called ___. a. capitalism b. communism c. socialism d. mercantilism

a. capitalism

The most socially responsible companies establish ____ for their vendors, setting clear policies for human rights, wages, safety, and environmental impact. a. Codes of Conduct b. Universal ethical standards c. Greenwashing principles d. Social responsibilities

a. codes of conduct

A(n) ______ is an example of a depository institution. a. commercial bank b. investment bank c. mutual fund d. insurance company

a. commercial bank

Which of the following is a key external user of a company's accounting information? a. competitors b. managers c. employees d. departmental managers

a. competitors

Andrew's parents want him to get a good education and, therefore, send him to the best school in the town. In this case, Andrew's education is the _____ his parents were looking for when they got him admitted in the school. a. core benefit b. augmented product c. implicit benefit d. pure product

a. core benefit

The nonprofit, nonpartisan Character Counts organization has worked with educators, community leaders, and experts to establish a set of: a. core ethical values that transcend political, ethnic, class, and religious differences. b. legal standards for ethical business behavior which all states are urged to adopt in order to reduce the complexity of ethical compliance. c. simple rules that can be used to solve common ethical dilemmas. d. ethical benchmarks that can be used to rate companies during a social audit.

a. core ethical values that transcend political, ethnic, class, and religious differences.

Companies typically engage in _______ to meet the needs of customers that don't have access to hard currency or credit. a. countertrade b. importing c. direct selling d. product churning

a. countertrade

The ___ is a project management tool that illustrates the relationships among all the activities involved in completing a project and identifies the sequence of activities likely to take the longest to complete. a. critical path method b. Gantt Chart c. program evaluation and review technique d. vertical integration technique

a. critical path method

A(n) ____ gives managers access to large amounts of data and the processing power to convert the data into high-quality information quickly and efficiently. a. decision support system b. intranet c. cybermediary d. expert system

a. decision support system

A(n) ___ is a decision support system that helps managers make better decisions in an area where they lack sufficient knowledge. a. expert system b. data mine c. graphical user interface d. utility system

a. expert system

Benjamin works for a private firm that has been contracted to examine the books of accounts of the Luigi Corporation. After checking the figures and examining the company's accounting methods, he is required to prepare a report on his findings. The work that Benjamin is tasked with is the characteristic of a(n) _____. a. external auditor b. management accountant c. risk analyst d. internal auditor

a. external auditor

Huge factories replaced skilled artisan workshops during the ____. a. industrial revolution b. entrepreneurship era c. production era d. marketing revolution

a. industrial revolution

The heavy reliance on accounting information by business stakeholders had led to it sometimes being called the: a. language of business b. cornerstone of business c. lifeblood of business d. standard of business

a. language of business

The ______ is a philosophy that makes customer satisfaction the central focus of an entire organization. a. marketing concept b. perceived value c. customer relationship management d. market segmentation

a. marketing concept

The stocks of firms that do not meet the listing requirements of stock exchanges are traded on the _______. a. over-the-counter market b. money market c. margin market d. private-player market

a. over-the-counter market

Marketing of raw materials, one of the business product categories, primarily emphasizes: a. price and service b. quality and brand c. customization d. product differentiation

a. price and service

Kyra Inc. has applied for a business loan in the United Bank. In order to best assess the loan case, the loan officer at the bank, Sirigu, decides to look at the company's net income figure. Sirigu will find this information in Kyra's _____. a. profit and loss statement b. budgeted income statement c. stockholders' equity statement d. statement of retained earnings

a. profit and loss statement

Which of the following must be done by human resource managers to nurture their human investment so that it yields the highest possible return? a. promoting career development b. recruiting employees for low wages c. encouraging employee lawsuit d. satisfying all demands of trade unions

a. promoting career development

Managerial accounting: a. provides reports and analysis to external stakeholders to show them the current business state. b. is the branch of accounting that adheres most strictly to the generally accepted accounting principles c. provides managers information about a firm's financial situation on a predetermined schedule. d. provides reports and analysis to managers to help them make informed business decisions

a. provides reports and analysis to external stakeholders to show them the current business state.

In the context of customer relationship management (CRM), a firm selling a high-ticket product to a small customer base is most likely to: a. pursue a full partnership with each of its key clients b. develop a limited relationship with its customers c. implement a soft-sell approach d. exclude its customers from the key aspects of the product development process

a. pursue a full partnership with each of its key clients

A clothing company, Patagonia, Inc., pledges one percent of its annual revenue to environmental causes around the world. Founders of the company believe they have an obligation to contribute to the world. This is an example of _____. a. social responsibility b. ethical relativity c. moral absolutism d. green washing

a. social responsibility

David is the chief executive officer of Grand Rocks mining company. He has convinced the company's board of directors to implement a change in the mining procedures which will reduce the company's impact on the environment in which it operates. This will ensure that the abilities of future generations to meet their needs will not be harmed. By agreeing to David's suggestion, Grand Rocks has embraced the idea of _____. a. sustainable development b. corporate philanthropy c. cause-related marketing d. planned obsolescence

a. sustainable development

The debt ceiling is: a. the maximum amount Congress lets the government borrow. b. the shortfall that occurs when expenses are higher than revenue over a given period of time. c. the overage that occurs when revenue is higher than expenses over a given period of time. d. the total amount of money within the national economy.1

a. the maximum amount Congress lets the government borrow.

Compared to a sole proprietorship, which of the following is considered an advantage of a general partnership?

ability to pool financial resources

There are about ________ people in the United States, but over ________ in the world.

290 million; 6 billion

The Dabble Corporation is a medium-sized firm that has decided to begin exporting its products. The company needs advice about how to get started as well as trade-finance support. The federal government has established _____________ to help companies in Dabble's position.

Export Assistance Centers

_______________ are specialists that match buyers and sellers from different countries and provide services to ease the process of entering global markets.

Export trading companies

Venture capitalists provide funds for small businesses as part of their philanthropy programs.

False

When writing a business plan, prospective entrepreneurs should remember to omit details regarding their experience and education since investors are only interested in the proposed business.

False

A social audit is a systematic evaluation of an organization's progress toward implementing programs that are profitable.

False Page 106

How is it that people working in their own self-interest produce goods, services, and wealth for others?

In order to earn money, people must sell some of what they produce to others.

According to the boxed material in Chapter 5 entitled "Dealing with Change," which of the following results from the wave of mergers and acquisitions that occurred during the last decade is most likely to occur?

Many of the merged firms will face difficulties and will end up "demerging."

Mini-Case Peppy Roaney believes that pizza is a much more versatile dish than most people realize. As the national marketing manager for Pizza Guys, he observed how consumers were delighted with barbecue pizza, Hawaiian pizza, Mexican pizza and other unique combinations. His suggestions for other unusual pizzas, however, were met with a negative response by his superiors. Even some of his colleagues ridiculed his ideas. Peppy's frustration has continued and he is now in the planning stages of starting his own business, Pizza Whirl. Peppy realizes that he will be taking a risk when he leaves his job with Pizza Guys and starts out on his own. However, Peppy is excited about the potential of profitably catering to the unmet desires of pizza consumers. Given Peppy's willingness to accept the risks of opening and running Pizza Whirl, he is clearly an example of a(n):

entrepreneur

The freedom of people to decide where they want to work and live, and what they want to buy or sell are basic capitalist rights under:

freedom of choice

According to the "Reaching Beyond Our Borders" box in Chapter 6, a tariff engineer is a person who:

has mastered the complicated tariff laws that cover the U.S. apparel trade.

Trans Globe Airlines has recently looked into a merger with Royal Blue Airlines, a financially troubled rival. The firms believe the merger will create a stronger company that can offer travelers more flights to a wider variety of destinations. This proposed merger is an example of a:

horizontal merger

A distinguishing feature of a cooperative is that it:

is owned and operated by the people who use it.

When a sole proprietor dies, the business:

may be taken over by the sole proprietor's heirs.

A Small Business Investment Company (SBIC) is a(n):

private investment company that is licensed by the SBA to make loans to small businesses.

Mary works a part-time job to put herself through college. She enjoys looking at fashion magazines and wishes she could afford the clothes that are advertised. Mary is not in the market for the designer clothes advertised in the magazines because:

she can't afford the clothes.

The amount of goods and services people can buy with the money they have is called their:

standard of living

A master limited partnership (MLP) is:

taxed like a partnership.

When preparing the executive summary section of a business plan, it is important to remember that:

the goal is to capture the attention of bankers and other investors who receive many business plans every day.

Which of the following statements about operating a U.S.-based franchise in a foreign country is most accurate?

The operating costs for franchises in foreign countries may be fairly high, but chances for success are quite good, because competition is likely to be less intense and the customer base in many foreign countries is expanding.

A firm desiring to enter a foreign market with a limited investment should consider foreign licensing

True Page 66

Joe Jackson operates a sole proprietorship, but he is in poor health and may be unable to continue running the business. If Joe becomes incapacitated, his business:

ceases to exist unless sold or taken over by Joe's heirs.


Related study sets

APES Unit 5 Progress Check MCQ Review

View Set

Forensic Accounting Final Exam Study Guide

View Set

Internet and Electronic Commerce

View Set

Personal Finance Quiz Chapter 1 ( Multiple Choice)

View Set